questions wrong

¡Supera tus tareas y exámenes ahora con Quizwiz!

A tourist from State A was severely injured in a bar fight in State B. The tourist filed a battery action against one of the bar's patrons, seeking $100,000 for his injuries. The defendant patron claims that the tourist is mistaken about who hit him. The patron says that he did not touch the tourist. The patron claims that it was the bar's bouncer-who looks like the patron-who hit the tourist and then continued to pummel him. The bouncer claims that he never touched or harmed the tourist. Can the patron assert a third-party claim against the bouncer to bring him into the action? A. No, because the patron has no legal basis to assert a claim against the bouncer and is not seeking to recover from the bouncer any portion of the patron's liability to the tourist. B. No, because the plaintiff is the master of his legal action, and defendants may not bring in new parties against whom the plaintiff has not asserted a claim. C. Yes, because the patron is claiming that the bouncer is liable for the injuries for which the plaintiff is suing the patron. D. Yes, because the patron's claims arise from the same transaction or occurrence as the plaintiff's original claims.

(A) The patron cannot assert a third-party claim against the bouncer to bring him into the action. Third-party claims may be asserted only to obtain recovery, and that recovery must be for the defendant/third-party plaintiff's own liability to the plaintiff. Since the patron is not seeking to recover from the bouncer any portion of the patron's liability to the tourist, he cannot assert a third-party claim.

A plaintiff is suing her former physician for slander, based on statements allegedly made by the physician to the plaintiff's co-worker that the plaintiff had difficulty conceiving children as a result of her history with venereal disease. The physician denies making any statements about the plaintiff. During the trial, the plaintiff calls as a witness her neighbor to testify that during her last visit to the physician's office, the physician told the neighbor that she was angry with the plaintiff for posting a negative review of her medical practice on a popular website, and that she made sure to "even the score" by sharing with others some opinions about the plaintiff. The physician objected to the testimony. How should the court rule on admissibility of the statement? A. Admissible, as an exception to the hearsay rule for a declaration against interest. B. Admissible nonhearsay. C. Inadmissible because of the physician-patient privilege. D. Inadmissible hearsay not within any exception.

(B) The court should find that the statement is admissible nonhearsay as a statement by a party-opponent (commonly called an admission). Under the Federal Rules, a statement made by a party and offered against that party is admissible nonhearsay. Here, the statement is admissible because it was made by the physician and is relevant because it directly conflicts with her contention at trial-that she did not make any slanderous statements about the plaintiff.

A defendant was charged with murdering his boss. After obtaining a valid search warrant and executing a valid search of the defendant's office, an officer found a love letter from the defendant's wife to his boss describing their sexual relations. The letter stated, "I can no longer hide my love for you from my husband. I intend to tell him about us and leave him for you." At trial, the officer seeks to testify about the contents of the letter as proof of the defendant's motive for killing his boss. The defense counsel should object on which of the following grounds? A. The letter was not authenticated as being from the defendant's wife. B. The testimony violates the best evidence rule. C. The testimony is hearsay not within any exception. D. The probative value of the evidence is substantially outweighed by the danger of unfair prejudice.

(B) The defense counsel should object because the officer's testimony violates the best evidence rule. Under the best evidence rule, the original writing must be produced when proving the material terms of that writing. Secondary evidence of the writing, such as oral testimony regarding the writing's contents, is permitted only after it has been shown that the original is unavailable for some reason other than by serious misconduct of the proponent. Here, the terms of the letter are material and no justification has been given for why it cannot be produced. Thus, the original letter must be produced and the officer's testimony violates the best evidence rule.

The plaintiff brought an action against a major national department store alleging that the electric blanket she bought from them overheated, causing a fire that destroyed her home and all that it contained. The defendant contends that its blanket could not have overheated unless it was left on after the plaintiff left for work on the day of the fire. The plaintiff offers in rebuttal the testimony of her husband, who will state that he has been married to the plaintiff for seven years, that he has slept in the same bed with her for most of that period, and that the first thing she does every morning upon awakening is to turn the control on the electric blanket to "off." Should this testimony be admitted? A. Yes, because prior conduct may be used to show conformity with habit. B. Yes, because evidence of habit may be used to show that a person acted in conformity with the habit on a particular occasion. C. No, because habit may only be established by opinion or reputation evidence, not specific conduct. D. No, because there is no corroboration of the husband's testimony by a nonparty witness.

(B) The husband's testimony is admissible as evidence of habit. Habit describes one's regular response to a specific set of circumstances. Evidence of a person's habit may be admitted to prove that, on a particular occasion, the person acted in accordance with the habit. [Fed. R. Evid. 406] Testimony that, every morning for the past seven years, the plaintiff has turned off the electric blanket immediately upon awakening describes her regular response to a repeated specific situation. Thus, the husband's testimony is evidence of habit. Such evidence is relevant to show that, on the day of the fire, the plaintiff's conduct was in conformity with this habit (i.e., that she turned off the blanket before leaving for work). Therefore, the husband's testimony should be admitted. (A) misstates the rule as to admissibility of habit evidence. Evidence of habit is used to show that particular conduct conformed with such habit. However, evidence of prior conduct is not used to show conformity with habit. The husband's testimony is admissible because it is evidence of what the plaintiff probably did the morning of the fire, not because it is evidence of what she did on prior occasions.

Concerned about the rising death toll on the state's highways, a state legislature enacted a statute providing for a summary one-year suspension of the driver's license of any person convicted of three speeding violations within a 12-month period. The statute provided that an administrative hearing is immediately available upon request. However, that hearing is limited to a determination of whether the licensee is the same person who was convicted of the speeding violations. A driver received three speeding citations in a three-week period and was convicted of all three charges. Her license was promptly suspended under the authority of the state statute. Without first seeking an administrative hearing, the driver files a suit in federal district court challenging the constitutionality of the statute. Should the court uphold the constitutionality of the state law? A. Yes, because driving an automobile on the state's highways is a privilege and not a right. B. Yes, because the state's interest in promptly removing unsafe drivers from its roads outweighs the driver's right to a prior hearing under these circumstances. C. No, because the law creates an irrebuttable presumption that all drivers falling within the ambit of the statute are unsafe. D. No, as a denial of due process without a prior hearing.

(B) The state law is valid because the prior judicial determinations that the driver violated the speeding laws satisfy the procedural due process requirements of the Fourteenth Amendment. Under the Due Process Clause of the Fourteenth Amendment, the state must provide some fair process or procedure before it may deprive a person of "life, liberty, or property." Fair procedure at a minimum requires an opportunity to present objections to the proposed action to a fair, neutral decisionmaker. Whether a prior evidentiary hearing is required and the extent of procedural requirements is determined by weighing (i) the importance of the individual interest involved, (ii) the value of specific procedural safeguards to that interest, and (iii) the governmental interest in fiscal and administrative efficiency. [Mathews v. Eldridge (1976)] Because the government has taken control of who may drive automobiles on public roads, which is a sufficiently important area of human activity that persons have a liberty interest in it, the government must provide fair procedure to those who are specifically barred from engaging in the activity. In applying the Mathews v. Eldridge balancing test, the Court has held that the state generally must afford a prior hearing before a driver's license is suspended or terminated. However, where the suspension is based on prior judicial determinations that traffic laws were violated, the driver has already had prior evidentiary hearings before unbiased decisionmakers on the significant factual issues involved. The governmental interest in keeping unsafe drivers off public roads and in not relitigating issues already fairly decided outweighs the driver's interest in keeping her driver's license. The procedural safeguards in the judicial proceedings in which she was convicted were sufficiently broad so that no additional prior hearing is necessary. Thus, the court should rule that the state law satisfies procedural due process requirements

The National Park Service recently created a new personnel level for field employees, which became the highest salaried position available to Park Service field employees. The position is restricted to employees over six feet in height. A female ranger who is five feet, three inches tall seeks your advice as to whether she can challenge the validity of the height restriction in federal court. If you decide to file suit on her behalf, which of the following would be your strongest argument against the validity of the restriction? A. Because most women are less than six feet tall, the restriction is unconstitutional as a violation of the Equal Rights Amendment. B. Because most women are less than six feet tall, the restriction is an invalid discrimination on the basis of gender in violation of the Due Process Clause of the Fifth Amendment. C. Because most women are less than six feet tall, the restriction is an invalid gender-based discrimination in violation of the Equal Protection Clause of the Fourteenth Amendment. D. The restriction denies the ranger a property right without an opportunity for a hearing before a neutral decisionmaker, in violation of the Due Process Clause of the Fifth Amendment

(B) The ranger's strongest argument, although by no means guaranteed of success, is that the height restriction is a gender-based classification that is not substantially related to important governmental interests. The Due Process Clause of the Fifth Amendment protects against action by the federal government. Although not expressly stated, this clause also provides an equal protection guarantee against federal action that generally applies to a similar extent that the Fourteenth Amendment Equal Protection Clause applies to the states. If the ranger can show that the Park Service restriction actually establishes a classification of eligibility for the new position based on gender, then the restriction will be found to violate the Fifth Amendment Due Process Clause unless the government has an exceedingly persuasive justification that the restriction is substantially related to important governmental interests. However, if the ranger is only able to show that the restriction has a discriminatory impact without being able to prove discriminatory intent, the court will not treat it as a gender-based classification and the ranger will not be successful.

A defendant was arrested and charged with selling narcotics. After he was given Miranda warnings, he said: "I am not talking and I want my lawyer!" He placed a call to his attorney who told him: "Do not talk until I get there!" The defendant was then placed in a cell with an undercover informant who had been instructed to try to get the defendant to talk about a recent murder in the area. While in the jail cell, the informant started talking about the murder, which the defendant had, in fact, committed. After a few hours of listening to the informant's incorrect story about the crime, the defendant got agitated and he hissed: "That's not where the gun is hidden, you moron!" A police officer standing nearby heard the defendant and told the detectives what he had heard. The defendant was removed from the cell and again given Miranda warnings. The defendant did not respond to the Miranda warnings. During questioning by the police about the murder, the defendant again got agitated and stated: "I'm not talking, and that's not where the gun is hidden!" The defendant's attorney finally arrived after his client was charged with murder. At his trial for murder, the defense moved to suppress the defendant's first statement made to the informant. How should the court rule? A. Grant the motion, because the defendant's Fifth Amendment right to counsel was violated when the police placed an informant in his cell. B. Grant the motion, because the defendant's Sixth Amendment right to counsel was violated. C. Deny the motion, because the defendant's Fifth Amendment rights were not violated. D. Deny the motion, because the defendant's statement to the police is admissible, so the admission of the first statement to the informant will not prejudice the defendant.

(C) The best answer is that the defendant's Fifth Amendment rights were not violated. The Fifth Amendment provides that no person "shall be compelled to be a witness against himself." To protect this right, the Supreme Court requires detainees to be informed of certain rights (that is, Miranda warnings) prior to custodial interrogation by someone the detainee knows is a police officer. The rationale is that the warnings help mitigate the inherent coercive nature of custodial police interrogation. Among the rights are the right to remain silent and the right to counsel. If a defendant invokes those rights, all questioning must cease. Here, there is no question the defendant was in custody, as he was arrested. There also is no question that he was given Miranda warnings and clearly claimed his right to remain silent and to counsel. However, his rights under Miranda were not violated because the defendant was questioned by an informant who the defendant did not know was working for the police. Therefore, there was no coercive interrogation in violation of the Fifth Amendment

As a result of an automobile accident at an intersection, the plaintiff sued the defendant, claiming that the defendant's car was traveling at a high rate of speed and went through a red light just before the crash. A witness for the plaintiff testified that he observed the accident and that the plaintiff's car was traveling at a low speed with a green light at the time of the accident. Which of the following will the court find NOT admissible to admit to impeach the credibility of the witness? A. A certified copy of a certificate of conviction for felony assault and battery seven years ago. B. The testimony of the witness's friend that, last month, while having a drink at a bar, the witness told her that the plaintiff's light was red. C. A record of an arrest one week ago for embezzlement. D. On cross-examination of the witness, the question, "Isn't it a fact that you lied to your employer last year concerning your meal expenses on a business trip?"

(C) A record of an arrest, even for a crime such as embezzlement, cannot be used to impeach the credibility of a witness. Since there was no criminal conviction, this would be classified as evidence of a prior bad act that demonstrates dishonesty. Federal Rule of Evidence 608 would allow an inquiry into such a prior bad act during cross-examination of the witness being impeached, but extrinsic evidence of such acts is not allowed under Rule 608, even if the witness denies the act on cross-examination.

A homeowner decided to destroy his home by fire to collect the insurance money. To work up his courage, he had several drinks at a local bar. When he returned to his block that night, he was so intoxicated that he mistakenly believed that his neighbor's house, which was 20 feet to the right of his house and looked very similar, was his own house. He started a fire under the back porch and went off a short distance to watch it burn. Suddenly he realized that he had the wrong house. He ran back and grabbed a garden hose and was able to put out the fire with just some slight charring of the porch. If the homeowner is charged with arson in a jurisdiction retaining the common law rules, what is the likely verdict? A. Not guilty, because he did not have the requisite intent to burn the dwelling of another. B. Not guilty, because he realized his mistake before any burning of the dwelling occurred. C. Guilty, because he acted with malice. D. Guilty, because his intent to commit arson of his own house is transferred to his neighbor's house.

(C) The homeowner should be found guilty. Common law arson consists of the malicious burning of the dwelling of another. At common law, the state of mind required-malice-is satisfied not only by intentionally burning the dwelling of another but also by acting with reckless disregard of an obvious risk that the structure would burn. While many courts ordinarily require that the defendant be subjectively aware of the risk, they will find malice when the failure to be aware of the risk is due to voluntary intoxication. Even had the homeowner done what he intended, he would have put his neighbor's house in jeopardy of burning. The fact that his intoxication caused him to fail to recognize the risk would not be a defense. Nor could he raise a mistake of fact defense because mistake of fact must be reasonable to negate the existence of malice, and here the facts state that his mistake was caused by his intoxication. (A) is incorrect because the malice required for common law arson may be satisfied by something less than the intent to burn down the dwelling of another, and here malice is established. (B) is incorrect because he caused a burning of the back porch, which is part of the dwelling, with the requisite malice; his conduct once he realized his mistake is irrelevant to his guilt. (D) is incorrect because his intent to burn down his own house does not constitute an intent to commit arson, which at common law is the burning of the house of another. His intent to burn down his own house also constituted malice for purposes of the burning of his neighbor's house, but not because of the doctrine of transferred intent.

A state legislature enacted a program by which students in the public schools could request instruction as to specific religions and religious beliefs, and thus participate in public school programs in which leaders of the religions involved gave religious instruction and performed religious practices on school grounds. The program provided instruction on any religion requested by a student. Which of the following would NOT be relevant in assessing the constitutionality of the state religious instruction program? A. The substantial effect of the legislation is to promote the religions studied. B. The primary purpose of the statute is to foster belief in the religions studied. C. The state does not have a compelling interest in instructing public school students about specific religions. D. The legislation requires that religious leaders and school officials interact constantly and frequently

(C) The statement in (C) would not be relevant in assessing the program's constitutionality. The facts in (A), (B), and (D) would make the program unconstitutional under the Establishment Clause of the First Amendment. A state law may not respect the establishment of a religion, and generally will be unconstitutional if it fails to pass any of the three following tests: (i) it has a secular purpose; (ii) its primary effect neither advances nor inhibits religion; and (iii) it does not produce excessive government entanglement with religion. [Lemon v. Kurtzman (1971)] Here, (A) is relevant to test (ii); (B) is relevant to test (i); and (D) is relevant to test (iii). The statement in (C) is irrelevant; the fact that no compelling interest exists would be relevant only if the state program had discriminated among religious sects.

A dentist filling a child's cavities used a newly developed local anesthetic that was more effective than Novocain. However, it carried a 1% risk of causing a serious seizure when administered to children, which the dentist did not mention to the child's mother or note in the consent forms, which stated only that a local anesthetic would be used. The child's dental work was completed without any problem, but the mother looked up the anesthetic on the Internet and learned about the risk. She complained to the dentist that she would not have consented to use of the anesthetic had she known of the risk, but the dentist argued that using the new anesthetic was justified in the child's case because otherwise he would not have been willing to sit still for the dental work. Does the mother have a cause of action on behalf of the child against the dentist? A. Yes, because a reasonable person would have considered information about the risk important. B. Yes, because the mother would not have consented to the use of the anesthetic if she had known of the risk of seizure. C. No, because the dentist used his best judgment in deciding that the benefits of using the anesthetic outweighed the risk. D. No, because the child suffered no harm from use of the anesthetic

(D) The mother has no cause of action because the child suffered no damages from the dentist's breach of duty. One of the duties that doctors, dentists, and other health professionals owe their patients is the duty to provide a patient with enough information about the risks of a proposed course of treatment or surgical procedure to enable the patient to make an "informed consent" to the treatment. If an undisclosed risk was serious enough that a reasonable person in the patient's position would have withheld consent to the treatment, the health care professional has breached this duty. However, breach of duty is only one element of a cause of action for negligence. The plaintiff must also establish actual and proximate cause and some damage to plaintiff's person or property. Damage means actual harm or injury. Unlike for some intentional torts, damage will not be presumed and nominal damages are not available. Note that complete absence of consent to a medical or surgical procedure may often constitute battery, which does not require damage as an element. However, a nondisclosure of the risks of the procedure is characterized instead as a breach of the duty of care. Here, the mother consented to the surgery and use of a local anesthetic, so battery is not applicable. Further, the child's dental work was completed without any problem and no other injury is apparent from the facts; the mother's possible distress at not being informed of the risk is not, standing alone, a compensable injury. [Restatement (Second) of Torts §436A] Hence, the mother does not have a cause of action against the dentist.

A defendant was charged with robbery and felony murder based on a death that arose during the robbery. The defendant pleaded not guilty and insisted on a jury trial. Right before the trial began, he fired his attorney and decided to defend himself. The court made a finding that the defendant was competent to represent himself at trial. The defendant then insisted on trying both of his charges separately in two different trials. The trial judge asked the defendant if he was confident that he wanted to have two separate trials. The defendant replied: "I am, your Honor." The felony murder case was tried first, and the jury found the defendant not guilty. The defendant then moved to dismiss the robbery charge based on double jeopardy. How should the court rule on the defendant's motion? A. Grant the motion, because double jeopardy forbids a second trial relitigating a lesser included offense. B. Grant the motion, because the defendant's Sixth Amendment right to counsel was violated. C. Deny the motion, because robbery is not a lesser included offense of felony murder. D. Deny the motion, because the charges could have been tried together but the defendant consented to the separate trials. The court should deny the motion.

(D) Where charges can be tried in a single trial, but the defendant consents to having two separate trials, there is no double jeopardy violation. Just as consent is an exception to the warrant requirement, consent to a second trial is an exception to the double jeopardy rule

An employee worked as a third-shift supervisor at a manufacturing plant. One of his duties was to ensure that all timekeeping records accurately reflected the time his crew actually worked. Workers, including the employee, were then paid for whatever hours the timecards reflected. The employee was also required to assist in submitting budgets for payroll. Needing to leave work early for a second job that he obtained, the employee had one of his trusted co-workers punch his card out at the regular time every day of the week. At the end of the week, he signed the timecard with those hours included, and was paid accordingly. He continued to do this for several weeks before being discovered. What crime has the employee committed? A. Forgery. B. Embezzlement. C. Larceny by trick. D. False pretenses.

(D) The employee has committed theft by false pretenses. At common law, theft by false pretenses occurs when a defendant (i) obtains title; (ii) to the property of another; (iii) by an intentional (or knowing) false statement of past or existing fact; (iv) with the intent to defraud another. In the instant case, the employee's conduct meets all of the elements of the crime. The employee obtained title to the property by falsely misrepresenting the number of hours that he worked, with the intent that the company would pay him for the hours. Thus, he has committed the crime of false pretenses. (A) is incorrect. At common law, forgery consisted of (i) a making or altering; (ii) of a false instrument; (iii) with the intent to defraud. The falsity must be about the instrument itself, not about the contents of the instrument. Here, the timecard is exactly what it purports to be-a timecard containing the hours worked. Thus, the employee has not committed forgery by submitting a false timecard. (B) is also incorrect.

A developer contracted with a general contractor to build an office building, and completion of the building was two years late. The developer filed a breach of contract action in federal district court against the general contractor, seeking damages caused by the delay. The general contractor filed a third-party claim against a major subcontractor, claiming that the subcontractor caused any delay and should be liable to the general contractor for anything the general contractor has to pay the developer. The subcontractor believes that the developer interfered with the subcontract and that the developer's interference caused not only the delay but also substantial cost overruns for the subcontractor. May the subcontractor assert a claim in the pending action against the developer seeking payment for the cost overruns? A. No, because the subcontractor is a third-party defendant and may not assert claims against the original plaintiff. B. No, because the subcontractor's claim does not seek indemnity for its liability to the general contractor, so the subcontractor may not assert this claim against the developer as an impleader claim. C. Yes, because the subcontractor and the developer are already parties to the action, and the subcontractor's claim arises from the same transaction or occurrence as the developer's original claim, so the subcontractor will be barred from asserting the claim in an independent action. D. Yes, because the subcontractor's claim against the developer arises from the same transaction or occurrence as the developer's original claim, but the subcontractor may assert the claim in an independent action if it prefers.

(D) The subcontractor may assert a claim against the developer in the pending action. A third-party defendant may assert a claim against the plaintiff if the claim arises out of the same transaction or occurrence as the plaintiff's original claim. (A) is therefore incorrect. Because the subcontractor's claim seeking payment for cost overruns and the developer's original contract claim arise out of the same transaction or occurrence, the subcontractor may assert its claim against the developer. (B) is incorrect because the subcontractor's claim does not have to seek indemnity; it must arise out of the same transaction or occurrence as the original claim. (C) is incorrect because a third-party defendant's claim against the plaintiff is not compulsory. Therefore, failure to assert the claim in the pending action would not bar the subcontractor from asserting it in an independent action.

A landowner owned two heavily wooded adjoining parcels of land containing a number of lakes. She conveyed the eastern parcel, which contained a hunting resort, consisting of a number of rental cabins, to a neighbor. The deed transferring the parcel also granted to "the neighbor, his heirs and assigns, and to invited guests of the resort all hunting rights and use of the woods on the western parcel for the benefit of the resort." Subsequently, the neighbor assigned his hunting rights to a hunter. When the landowner discovered the hunter hunting on her land, she brought an appropriate action to declare his rights void. If the court rules for the landowner, it will be because the neighbor's right to hunt on the western parcel is: A. A profit appurtenant. B. A profit in gross. C. An easement in gross. D. A license.

(A) (A) If the court rules for the landowner, it will be because the neighbor's right to hunt and fish on the western parcel is a profit appurtenant. A profit is a nonpossessory interest in land that entitles the holder of the profit to enter on the servient tenement and take something off of the land (e.g., minerals, timber, oil, or game). Like an easement, a profit may be appurtenant or in gross. If the profit exists to serve a dominant estate, the profit is appurtenant and can only be transferred along with the dominant estate. Conversely, if the profit does not exist to serve a dominant estate, it is a profit in gross and may be transferred separate and apart from the dominant estate. Here, the neighbor has a profit with respect to the game on the western parcel. Because the profit is "for the benefit of the resort" on the eastern parcel, it is appurtenant rather than in gross because it serves the dominant estate (the eastern parcel). Thus, the neighbor's assignment of the profit to the hunter is void.

A computer programmer sent a computer virus anonymously via e-mail to a business. The programmer believed that the virus would just disable the business's e-mail program for a short time without causing any additional damage, although he was aware that it very infrequently caused widespread damage to the infected computer. However, because of a hidden bug in the business's e-mail program, the virus infected the computer's entire hard drive, eventually rendering it unusable. Not only did the business lose important data, it also had to replace the computer, at a cost of over $1,000. The jurisdiction in which this occurred has a modern criminal code patterned after the Model Penal Code. One of its statutes makes it a criminal offense to "knowingly cause over $200 in damage to another's property." May the programmer be found guilty of violating the statute? A. No, because the programmer did not know that the virus would cause damage to the computer's hard drive. B. No, because the programmer did not intend to cause the damage to the computer's hard drive. C. Yes, because the programmer knew that he was sending a virus to the business's e-mail program. D. Yes, because the programmer was aware that in a very small percentage of cases the virus could cause widespread damage to a computer system.

(A) The computer programmer cannot be found guilty of violating the statute because he did not know that his act would cause the damage to the business's computer that it did. Under the Model Penal Code fault standards adopted by modern criminal codes, a person acts "knowingly" with respect to the nature of his conduct when he is aware that his conduct is of that nature or that certain circumstances exist. He acts knowingly with respect to the result of his conduct when he knows that his conduct will necessarily or very likely cause such a result. When a statute establishes a culpable state of mind without indicating to which material elements of the offense it is to apply, the statute will be interpreted as requiring that state of mind for every material element of the offense. In this case, the statute requires that the defendant "knowingly cause over $200 in damage to another's property." The requirement that the damage caused be over $200 is a material element of the offense because it defines the harmful result that will trigger criminal liability under the statute. Thus, the programmer must have known that his act of sending the computer virus would necessarily or very likely cause over $200 in damage to the business's computer to be liable under the statute in this case. (B) is incorrect because intent is not required by the statute for the programmer to be liable. Under modern criminal codes, intent is equated with purpose, which is defined as having a conscious objective to engage in certain conduct or cause a certain result. Here, the programmer could be guilty under the statute even if he did not have the objective of causing that damage to the business's computer, as long as he knew that it was at least very likely to occur. (C) is incorrect because the fact that the programmer knew that he was sending a virus is not enough to establish guilt. As discussed above, the statute also requires that he know that his conduct will or is very likely to cause over $200 in damage to the business's computer. (D) is incorrect because, even if the programmer knew that there was a small chance that this damage might occur, he has not acted with the required degree of culpability under the statute. The programmer must have known, at a minimum, that his conduct was very likely to cause the damage to the business's computer. Being aware that such damage could occur in a very small percentage of cases may establish that the programmer acted recklessly, but it does not establish that he acted knowingly.

A state statute allows for criminal trials by a jury composed of six jurors. Five of the six jurors must concur for a guilty verdict. The defendant is charged with petty larceny, which carries a maximum sentence of one year's imprisonment, plus a fine of $2,500. Before voir dire begins, the defendant objects to both the six-member jury and the fact that only five of six jurors are needed for a conviction. Should the trial judge overrule the objection? A. No, because the use of a six-person jury is unconstitutional for this offense. B. No, because the number of jurors needed for a conviction is unconstitutional for this offense. C. Yes, because the right to a trial by jury does not constitutionally require a jury of 12. D. Yes, because a state is permitted to set its own jury trial requirements for petty offenses.

(B) The judge should not overrule the objection. There is no constitutional right to a jury of 12, but there must be at least six jurors to satisfy the right to a jury trial under the Sixth and Fourteenth Amendments. Also, the Supreme Court has held that juries must be unanimous. Thus, the fact that a six-member jury was used would be constitutionally permissible, making (A) incorrect, but the requirement that only five of six jurors must concur for a guilty verdict would not be constitutionally permissible.

One provision of a federal law provided that state governments may enact legislation regulating any form of pinball machine or video game, including location and hours of operation. In response, a Western state enacted legislation providing, among other things, that any video game sold or operated within the state use a particular LCD screen designed to minimize eyestrain. A corporation that designs and manufactures video games for sale throughout the United States and in Europe is based in the Midwest. Approximately 10% of its gross sales are made in the Western state that has regulated the LCD screens. The corporation's machines are not manufactured using the special eye-protecting LCD screens; to install such screens in all machines manufactured would cause the price of the machines to increase by 20%, and to use the screens in machines sold only in the Western state would increase the cost of those machines by 50%. T he corporation files suit in federal court to enjoin enforcement of the state video game statute. How should the court rule? A. For the state, because the challenged legislation is within the powers specifically reserved to the states by the Tenth Amendment. B. For the state, because Congress has acted within its power to authorize video game regulation by the states. C. For the corporation, because the challenged statute violates the Commerce Clause. D. For the corporation, because the challenged statute is overbroad and exceeds the permissible bounds of regulation as authorized by Congress.

(B) The state will prevail because Congress has consented to the state regulation of interstate commerce. A state may regulate local aspects of commerce (i.e., intrastate commerce), but state regulation that discriminates against or substantially burdens interstate commerce may be held invalid under the Supremacy Clause, because of Congress's very broad power to regulate interstate commerce under the Commerce Clause. Here, the state statute, standing alone, might have been held invalid because its substantial burden on interstate commerce could have been found to outweigh any legitimate local interest in reducing eyestrain. However, the federal statute changes the equation. Because Congress's power over interstate commerce is very broad, Congress may allow a state to adopt legislation that would otherwise be invalid as an unconstitutional burden on interstate commerce; this is what Congress did here. By allowing the state regulation, it is actually exercising the federal commerce power-it simply allows for nonuniform (state-by-state) rules. Therefore, (B) is correct and (C) is incorrect. (A) is incorrect because the Tenth Amendment merely reserves to the states the powers not delegated to the federal government, and the power to regulate interstate commerce-the power in question here-was not reserved to the states because it was granted to Congress by the Constitution. (D) is incorrect because the statute, as far as described in the problem, appears to be exactly what Congress authorized. There is no information indicating that the statute is overbroad or exceeds the authorization, which apparently allows a broad range of regulation beyond just location and hours of operation.

A state statute has adopted the common law definition of larceny. Another statute provided as follows: "It shall be an affirmative defense to a crime if the defendant establishes by clear and convincing evidence that, due to a mental disease or defect, he was unable to appreciate the criminality of his conduct or conform his conduct to the requirements of the law." A homeowner was leaving town for two weeks and he asked his neighbor to stop by the house each day and water the plants. While at the homeowner's home, the neighbor found the keys to the homeowner's new car. The neighbor took the car and drove it into town to show his friends. The neighbor told all of his friends that he had purchased the car. The homeowner returned home three days early, saw that the car was missing, and called the police. Later that day, the neighbor was arrested and charged with larceny. At the neighbor's trial, the neighbor testified that he intended to return the car. Additionally, two psychiatrists testified that, due to a mental defect, the neighbor suffered from an extreme inferiority complex and delusions of grandeur. The doctors further testified that his mental condition caused him to take the car and to tell other people that he owned it. At the conclusion of the evidence, the court's instructions to the jury included the following: 1. If you find by a preponderance of the evidence that the defendant intended to return the car, you should find the defendant not guilty. 2. If you find by a preponderance of the evidence that, due to a mental disease or defect, the defendant was unable to appreciate the criminality of his conduct or conform his conduct to the requirements of the law, you should find the defendant not guilty. The neighbor was found guilty and he appealed, claiming that the jury instructions violated his constitutional rights. How should the appellate court rule? A. Both instructions were constitutional. B. Both instructions were unconstitutional. C. Instruction 1 was unconstitutional; Instruction 2 was constitutional. D. Instruction 1 was constitutional; Instruction 2 was unconstitutional.

(C) The appellate court should rule that only Instruction 2 was constitutional. Due process of law requires a state to prove each element of the crime charged beyond a reasonable doubt. However, as to affirmative defenses to the criminal charge, the Supreme Court has held that the state can place the burden of proof on the defendant without violating the defendant's constitutional rights. [Leland v. Oregon (1952)] Common law larceny is the taking and carrying away of property in the possession of another with the intent to permanently deprive the other of the property. Since the intent to permanently deprive is an element of the crime, the state cannot require the defendant to prove that he intended to return the car, which would negate the required intent for larceny. The state must prove beyond a reasonable doubt that he intended to permanently deprive. Hence, Instruction 1 is unconstitutional, making (A) and (D) wrong. Instruction 2 is constitutional. As to the defense of mental illness, the Supreme Court has held that the state can require (as federal courts do) that the defendant prove the defense by clear and convincing evidence. The state also can place the burden on the defendant to prove the defense of mental illness by a preponderance of the evidence, which is a lesser burden of proof. The fact that the instruction placed a lighter burden on the defendant than the statute specified would not make the instruction a violation of the defendant's constitutional rights. Thus, choices (B) and (D) are wrong.

A father conveyed his property to his son and daughter "as joint tenants with right of survivorship, but if they ever attempt to sell the property during their lifetimes, a right of first refusal based on the sale price is hereby granted to my sister." The deed was promptly and properly recorded. Unbeknownst to the son or the sister, the daughter quitclaimed her interest in the property to a purchaser. The following month, the daughter was killed in a snowmobile accident. The purchaser of the daughter's interest filed a suit for partition of the property. The son filed an appropriate counterclaim for quiet title, asserting that he was the owner of the entire parcel. The sister also filed a counterclaim, asserting that her right of first refusal was valid and that she was prepared to exercise her right to purchase the property for the contract price. In a jurisdiction in which the Rule Against Perpetuities is unmodified by statute, how should the court rule? A. For the purchaser, because the right of first refusal is invalid as an unreasonable restraint on alienation. B. For the son, because he succeeded to the entire ownership when the daughter died. C. For the son, because the right of first refusal violates the Rule Against Perpetuities. D. For the sister, because she has a valid right of first refusal.

(D) The court should rule for the sister because she has a valid right of first refusal that she is entitled to exercise. A joint tenancy is a type of concurrent ownership of a parcel of land that is distinguished primarily by the right of survivorship; i.e., when one joint tenant dies, the property is freed of her concurrent interest and the survivor retains an undivided right in the property that is no longer subject to the interest of the deceased co-tenant. To create a joint tenancy, a grantor must explicitly indicate in the conveyance that the parties are to hold as joint tenants. If one joint tenant conveys her interest, the joint tenancy is severed. The new tenant holds as a tenant in common with the remaining joint tenant, so that there is no longer a right of survivorship. Here, the father, the grantor of the property, expressly indicated in the conveyance that the son and the daughter were to take the property as joint tenants with right of survivorship. When the daughter quitclaimed her interest in the property to the purchaser, the joint tenancy between the daughter and the son was severed (despite the fact that the son did not know of the conveyance to the purchaser). However, the daughter's conveyance triggered the sister's right of first refusal. A right of first refusal gives its holder the preemptive right to meet any third party's offer to purchase real estate. Thus, the sister should have been given the opportunity to meet the purchaser's offer. Because the purchaser's interest in the property is no greater than the daughter's interest, the sister may enforce her right of first refusal against the purchaser. (A) is incorrect because the right of first refusal is a valid restraint on alienation if it is reasonable. Here, the right of first refusal is based on the sale price of the property, which would be deemed reasonable.

On the last play of a playoff football game, a game-winning touchdown was nullified by a questionable penalty called by the referee. To register her displeasure but without intending to hit anyone, a fan sitting in the stands threw a bottle onto the field that just missed the head of the referee, who was looking in the other direction and did not see the bottle being thrown. The fan was charged with assault. Should the fan be convicted? A. Yes, because throwing the bottle was a substantial step towards commission of a battery. B. No, because the referee did not see the bottle. C. No, because the fan did not intend to hit anyone. D. No, because the referee did not see the bottle, nor did the fan intend to hit anyone.

(D) The fan should not be convicted under either definition of assault. Criminal assault is either: (i) an attempt to commit a battery, or (ii) the intentional creation, other than by mere words, of a reasonable apprehension in the mind of the victim of imminent bodily harm. In the instant case, the fan did not intend to hit anyone. Thus, the fan's actions do not constitute an attempt to commit a battery, which would require a specific intent to bring about bodily injury or an offensive touching. In addition, the fact that the referee did not see the bottle being thrown at him means that the defendant did not create in the referee a reasonable apprehension of imminent bodily harm. Consequently, the fan is not likely to be convicted of either type of assault.

A kidnapper and his cohort hatched a scheme to kidnap the son of a wealthy man and hold him for ransom. After conducting a surveillance of the wealthy man's home, they decided that they would have to have inside help to disable the alarm at the home. They agreed that the kidnapper would contact the man's butler, who they learned was heavily in debt and frequented a local racetrack during his time off. The butler would be offered money to disconnect the alarm on the night of the planned kidnapping. Shortly before the kidnapper was to go to the track to make contact with the butler, the cohort had a change of heart about the scheme and contacted the butler. He warned the butler not to have anything to do with the kidnapper. The butler met with the kidnapper anyway and pretended to go along with his proposal, accepting the down payment that the kidnapper offered. After meeting with him, the butler contacted the authorities. The kidnapper and cohort are charged with conspiracy in a jurisdiction that follows the common law rule for conspiracy. What is the most likely result? A. Both the kidnapper and cohort are guilty of conspiracy because the cohort agreed with the kidnapper to commit the offense. B. The cohort is not guilty of conspiracy because he withdrew from the conspiracy by contacting the butler. C. The cohort is not guilty of conspiracy because he withdrew from the conspiracy by contacting the butler, and the kidnapper is not guilty of conspiracy with the butler because one cannot be a conspirator by oneself. D. The kidnapper is guilty of conspiracy with the butler.

(A) Both the kidnapper and the cohort are guilty of conspiracy. At common law, a conspiracy was an agreement between two or more persons to commit an unlawful act or to commit a lawful act in an unlawful manner. The elements are: (i) an agreement between two or more persons; (ii) the intent to enter into an agreement; and (iii) the intent to achieve the objective of the agreement. When the kidnapper and the cohort decided to kidnap the wealthy man's son, they were guilty of common law conspiracy, which did not require an overt act in furtherance of the conspiracy. (A majority of states now require an overt act in furtherance of the conspiracy, but mere preparation, such as the surveillance here, suffices.)

In January, an owner executed and delivered a mortgage on her property to a bank to secure a $50,000 loan. Due to a clerical error, the mortgage was not recorded at that time. On February 15, the owner entered into a contract to sell the property to a buyer for $150,000. On February 16, the owner took out a $30,000 mortgage on the property with a finance company. The finance company promptly and properly recorded its mortgage. Knowing nothing about either of the mortgages, the buyer closed on the property on April 1, tendering $150,000 to the owner. The owner gave the buyer a warranty deed to the property. On April 3, the bank discovered its error and properly recorded its mortgage that same day. The buyer recorded his deed to the property on April 6. The jurisdiction in which the property is located permits mortgages on property under contract, and has a statute that provides: "No conveyance or mortgage of real property shall be valid against a subsequent purchaser for value and without notice whose conveyance is first recorded." The bank brings an appropriate action to determine the status of its mortgage on the property. What should be the court's determination? A. The buyer holds the property subject to both mortgages, and the bank's mortgage is subordinate to the finance company's mortgage. B. The buyer holds the property subject to both mortgages, and the bank's mortgage is superior to the finance company's mortgage. C. The buyer holds the property subject only to the finance company's mortgage. D. The buyer holds the property subject only to the bank's mortgage.

(A) The buyer holds the property subject to both mortgages, and the bank's mortgage is subordinate to the finance company's mortgage. The jurisdiction's recording act is a race-notice statute. Under this statute, a bona fide purchaser is protected only if he records before the prior transferee or mortgagee records. Here, the buyer had record notice of the finance company's mortgage, so he was not a bona fide purchaser protected by the recording statute as to that mortgage. As to the bank's mortgage, the buyer was a bona fide purchaser because he had no notice of that mortgage executed by the owner, but he did not record until after that mortgage was recorded. Hence, the buyer holds the property subject to both mortgages. The bank's mortgage is subordinate to the finance company's mortgage because mortgagees for value are treated as "purchasers" under the recording statutes, and the finance company executed its mortgage without notice of the bank's prior mortgage and recorded it before the bank recorded its mortgage. Thus, the finance company's mortgage has priority over the bank's mortgage,

A state statute prohibited the sale or possession of any food product containing more than one part per billion of a dangerous pesticide. An out-of-state driver taking her recreational vehicle through a corner of the state was stopped at a state inspection station. When the state trooper learned that the pantry of her RV was stocked with food, he asked to test a few samples of her baked goods. The samples contained about 600 parts per billion of the prohibited pesticide, and all of the other baked goods in her possession were tested and found to have the same level of pesticide. All of her baked goods, worth about $150, were confiscated and destroyed. The state in which the driver lived has no laws governing the pesticide level of baked goods. A federal law designed to protect agricultural workers requires that any food product containing more than 500 parts per billion of the toxic pesticide must be labeled as such and be in special containers. The driver brings an action in federal court asserting that the state statute is invalid because it is preempted by the federal law. How should the court rule as to this claim? A. For the state, because the purposes of the federal law are different from those of the challenged statute. B. For the state, because regulation of food quality is a power reserved to the states by the Tenth Amendment. C. For the driver, because the federal law does not expressly permit states to enact more stringent pesticide level controls. D. For the driver, because the federal law and the state statute regulate the same subject matter.

(A) The court should rule for the state because the purpose of the federal law is different from the purpose of the state law. The question here is whether the state law is preempted by the federal law. Implied preemption will be found where it was the intent of the federal government to occupy the entire field with its regulation, the state law directly conflicts with the federal law, or the state provisions prevent achievement of federal objectives. Because the federal law here is aimed only at occupational safety, no conclusion can be drawn that the federal government intended to occupy the entire field of regulation of pesticides, and the state law does not interfere with the federal law. For regulations involving health, safety, and welfare, the Court will presume that state police powers are not preempted unless that was the clear and manifest purpose of Congress when it enacted the federal law.

A homeowner borrowed $50,000 from a bank, secured by a mortgage on his home. Shortly thereafter, the homeowner sold his home to a buyer for $70,000 by a deed containing a recital signed by both parties that title passed "subject to" the bank's mortgage, "which obligation grantee expressly assumes." The buyer paid the homeowner $20,000, took possession of the house, and began making monthly payments of principal and interest to the bank. A few years later, a chemical manufacturing firm built a huge sulfur processing plant just down the road from the home, which caused the house to immediately decline in value to $35,000. Subsequently, the buyer stopped making the monthly payments to the bank. The bank exercised its contractual right of nonjudicial foreclosure and sold the house at a public auction for $34,000. The bank then brought suit against the homeowner and the buyer for $14,000, the difference between the proceeds of the foreclosure sale and the $48,000 principal remaining due on the original loan to the homeowner. The jurisdiction does not bar deficiency judgments. Against whom should the bank be granted a judgment for $14,000? A. Both the homeowner and the buyer. B. Only the homeowner. C. Only the buyer. D. No one.

(A) Both the homeowner and the buyer are liable for the deficiency. If a sale of foreclosed property does not bring enough to satisfy the mortgage debt, the mortgagee/lender can bring a personal action against the mortgagor/debtor for the deficiency (as long as the jurisdiction does not bar deficiency judgments). When the mortgagor sells the mortgaged property and gives a deed, the grantee takes subject to the mortgage, which remains on the land. If the grantee does not sign an agreement to assume the mortgage, he does not become personally liable on the loan, and the original mortgagor remains personally liable. If the grantee does sign an assumption agreement, however, the lender is considered a third-party beneficiary of the agreement, and hence may recover from the assuming grantee, who is primarily liable, or the original mortgagor, who is secondarily liable. Here, the buyer signed the recital providing for the assumption, so she will be personally liable on the loan.

A three-car accident occurred in which the drivers were a citizen of State A, a citizen of State B, and a citizen of State C. The State A citizen filed a negligence action against the other two in federal district court and lost his case. After judgment, may the State C citizen assert and maintain a negligence action against the State B citizen seeking damages for the injuries the State C citizen sustained in the same accident? A. Yes, because, while the State C citizen could have asserted the claim as a cross-claim in the prior action, he may wait and assert it as an independent action. B. Yes, because the State C citizen could not have asserted the claim in the prior action and thus may assert it independently. C. No, because the State C citizen's claim was a compulsory cross-claim in the prior action and, since it was not asserted as a cross-claim in that action, it is now barred. D. No, because the State C citizen's claim is barred by claim preclusion.

(A) The State C citizen may assert and maintain a negligence action against the State B citizen. The claim could have been asserted as a cross-claim in the prior action because it arose from the same transaction or occurrence, but cross-claims are never compulsory. (B) and (C) are therefore incorrect. (D) is incorrect because claim preclusion applies to cases brought by the same claimant against the same defendant. The State C citizen was a defendant in the first case and would be a claimant in the second. Therefore, claim preclusion would not bar the State C citizen's claim.

Auto workers went on strike in a town heavily reliant on the auto industry. While negotiations between the union and management were ongoing, a person intercepted and recorded a phone call between the union's president and management's chief negotiator. A state statute makes it illegal to record a phone call without the consent of the parties being recorded. The statute also makes it illegal to play an illegally recorded conversation on television or radio. The person who recorded the call anonymously sent the recording to a local TV station. The TV station news anchor played the recording on air. Can the anchor who played the recording be prosecuted under the statute? A. No, because the anchor did not record the conversation, and the information is truthful and about a matter of public significance. B. No, because once media organizations obtain information, the First Amendment gives them a right to publish it. C. Yes, because allowing publication of such recordings would encourage further violations of an otherwise valid law. D. Yes, because the television station should have brought an action to test the validity of the law before playing the recording on air.

(A) The anchor may not be prosecuted. Generally, the press has a right to publish information about a matter of public concern, and this right can be restricted only by a sanction that is narrowly tailored to further a state interest of the highest order. The right applies even if the information has been unlawfully obtained in the first instance, as long as (1) the speech relates to a matter of public concern, (2) the publisher did not obtain it unlawfully or know who did, and (3) the original speaker's privacy expectations are low. Here, because a strike is ongoing, the labor negotiation very likely is a matter of public concern, and the speakers could have anticipated great public interest in the conversation. Moreover, since each party to the conversation was speaking to an adversary, neither would have been justified in thinking the details of the conversation would be kept private. The anchor received the recording anonymously. Therefore, the anchor cannot be prosecuted.

A large insurance company instituted a supplemental benefit plan for its own employees. Under the plan, any employee who had worked for the company for at least 25 years would be permitted to designate a charity to receive, on the employee's retirement, a donation in the employee's name of six months' worth of the employee's salary. The plan gave participating employees an unqualified right to change the beneficiary at any time before payment was made. An employee nearing retirement enrolled in the plan and named his favorite church as the beneficiary of the donation. The church received a letter from the company informing it that the employee had named it beneficiary of his plan and indicating the approximate amount that it would receive upon the employee's retirement in 10 months. The letter did not inform the church of the employee's right to change beneficiaries before that time. Church elders, anticipating the gift, authorized restoration work to the church building, making plans to pay for the work with the funds from the employee's benefit program. Six months later, the employee converted to a different religion and changed the beneficiary of his plan to his new church. When the employee retired, the company paid the benefit to his new church. His old church, which had paid for the restoration work on its completion, demanded payment of the benefit from the company. When payment was refused, the church sued the company. Which party is likely to prevail A. The church, because the interests of justice require it. B. The church, because its rights as third-party beneficiary had vested when it was informed in writing that it was the beneficiary. C. The company, because the agreement between the employee and the company allowed the employee to change the beneficiary of the benefit plan. D. The company, because it had a duty to pay the employee's new church as the named beneficiary of his plan.

(A) The church will be able to recover against the insurance company because the interests of justice require it. Under the majority view, consideration is not necessary to make an agreement at least partially enforceable where the facts indicate that the promisor should be estopped from not performing. This is stating the concept of promissory estoppel without labeling it as such. Under the Second Restatement, a promise is enforceable to the extent necessary to prevent injustice if the promisor should reasonably expect the promise to induce action or forbearance and such action or forbearance is in fact induced. Here, the insurance company sent a letter to the church informing it that the employee had named the church beneficiary under his employee benefits program. The company did not warn the church that the employee had the right to change his beneficiary and should have reasonably expected that the church would rely on the promise in some way; it is not necessary in charitable contribution cases that the promisor know of a specific expenditure that the recipient made or is going to make. The church did in fact rely on the promise by authorizing and paying for the restoration work. Hence, to prevent injustice, as choice (A) states, the church can recover against the company to the extent of its reliance (the cost of the restoration work)

On February 3, a property owner mailed an offer to a married couple who had expressed an interest in buying his property at 337 Green Street. The offer asked for $200,000, "terms $60,000 cash, with the balance secured by a first mortgage." The offer reached the couple on February 5. On February 8, the couple replied by e-mail that the offer had been received and was being considered, and added, "We would much prefer a straight cash deal. Would you consider an immediate purchase for $180,000 cash?" On February 10, the property owner responded with a one-word e-mail: "No." After reading the property owner's response, on February 11 the couple e-mailed: "E-mail received. We accept your offer of February 3. Tender the deed c/o our agent, The First National Bank and Trust." The property owner now refuses to sell the property. If the couple sues for specific performance, are they likely to succeed? A. Yes, because the couple's February 8 e-mail was an inquiry. B. Yes, because the couple's response of February 8 operated as an acceptance. C. No, because the couple's response of February 8 operated to terminate the property owner's offer. D. No, because the communications fail to establish the terms of the proposed agreement with sufficient definiteness to be enforced through specific performance.

(A) The couple will likely succeed because the February 8 e-mail was a mere inquiry rather than a counteroffer. A contract was formed here because there was mutual assent and valid consideration. For there to be mutual assent, there must be a valid offer and an unequivocal acceptance before the offer is either rejected by the offeree or revoked by the offeror. Here, the property owner offered to sell his house and lot for $200,000. This was clearly an offer to the couple. The issue here is whether either of the couple's replies constitutes an unequivocal acceptance of this offer. A counteroffer serves as a rejection of the original offer as well as a new offer. However, a mere inquiry about additional terms or matters is not a counteroffer. The test of whether the reply is a counteroffer or inquiry is whether a reasonable person would believe that the offer was being rejected. Here, the couple's February 8 communication was a mere inquiry, rather than a counteroffer. The couple's statements do not show an outright rejection unless their terms are agreed to; they merely state that the offer was being considered and then ask the property owner to consider their proposal. The February 11 communication is an acceptance. The couple clearly state that they accept the "offer of February 3" (showing that they are agreeing to the property owner's original terms). The additional language in their e-mail (about the deed and their agent) is not an alteration of the original terms because implicit in a sale of land contract is that a deed will be conveyed, and the use of an agent is immaterial. Statements by the couple that make implicit terms explicit do not prevent acceptance of the offer. Thus, their second communication unequivocally accepts the property owner's offer. Because promises were exchanged (a promise to sell the property for a promise to pay $200,000), valid consideration existed, and a contract was formed.

A philanthropist told his friend, who was a state governor, that he planned to build a museum. The governor thought that the museum would bolster the state's tourism industry and offered to arrange to have the state purchase land and grant it to the museum to enable the philanthropist to build a bigger museum with his money than originally planned. The philanthropist agreed, and the museum was built. The philanthropist undertook the hiring of the museum's senior staff. He was of German descent and was ashamed of Germany's actions during World War II. To assuage his own conscience, he refused to hire anyone whom he believed to be of German descent. A restoration expert applied for a job as chief curator of the museum, but the philanthropist refused to hire him because of his German background. The restoration expert discovered the philanthropist's rationale and brings suit against the museum, claiming that the hiring practice violates his constitutional rights. How is the court most likely to rule? A. The policy is constitutional, because the museum is a private entity and so may constitutionally hire and fire as it desires. B. The policy is constitutional, to the extent necessary to remedy past discrimination. C. The policy is unconstitutional under the Equal Protection Clause, because the grant of the land is sufficient state involvement to render the museum's actions state action. D. The policy is unconstitutional under the Equal Protection Clause, because the state will benefit from the museum and this creates a sufficient nexus to find state action.

(A) The court should find that the museum is a private entity and that it may constitutionally hire and fire as it pleases because its actions do not constitute state action. The Equal Protection Clause prohibits states from discriminating against persons on the basis of race, alienage, or national origin unless the discrimination is necessary to achieve a compelling state interest. The museum's policy here of not hiring persons of German descent clearly violates the Clause's prohibitions. However, there is no constitutional violation here because there is no state action. The Equal Protection Clause prohibits only government infringement. This does not mean that only direct government action is proscribed. Private action may constitute state action where the private actor is performing an exclusive state function or the government is significantly involved in the private actor's activities. The running of the museum here, however, is not an exclusive government function (e.g., running elections), and the state's grant of the land for the museum does not constitute significant state involvement in the museum's affairs (see below). Thus, there is no state action here and no constitutional violation. (Note that the museum's actions probably violate several civil rights statutes that apply to private citizens.) (B) is incorrect because it reaches the proper result on a faulty rationale. If the museum's acts were state action, the excuse of remedying past discrimination would not validate the discrimination. Remedying past discrimination has never been used as a basis for explicit discrimination against a suspect class, but rather has only been used to justify action favoring a group in limited circumstances. As noted above, discrimination against a suspect class will be upheld only if it is necessary to achieve a compelling interest, and only one case of explicit discrimination against a suspect class has been upheld-incarcerating United States citizens of Japanese ancestry during World War II. It is doubtful that the explicit discrimination would be upheld. (C) is incorrect because the grant of land simply does not constitute significant state involvement, which requires that the state affirmatively facilitate, encourage, or authorize the acts of discrimination. Merely granting land to an entity that then decides to adopt a discriminatory policy does not constitute the necessary affirmative action.

A resort maintained an outside bar adjacent to its pool. When the bar was closed, it was secured by a metal gate that reached up towards the roof of the bar, but which left about a three-foot gap between the top of the gate and the roof. The resort had installed motion detectors inside the bar linked to an alarm system because of several previous thefts of liquor by persons climbing over the gate. Late one night, an intoxicated guest of the resort who wanted to keep partying after hours began to climb over the gate to get into the bar through the gap at the top, intending to take some bottles of wine. The brackets attaching the gate to the walls, which had been gradually deteriorating and pulling away from the walls for some time, suddenly gave way as he reached the top. The gate collapsed, causing him to fall back onto the concrete patio. He sustained a severe concussion and other serious injuries. The resort is located in a jurisdiction that applies the traditional liability rules for landowners and possessors of land. If the guest sues the resort for his injuries, is he likely to prevail? A. No, because the guest did not have invitee status when he was climbing over the gate. B. No, because the guest intended to steal alcohol belonging to the resort. C. Yes, because the resort operators were aware that persons had climbed over the gate in the past. D. Yes, because the brackets attaching the gate to the walls were in a weakened condition that could have been detected by a routine inspection.

(A) The guest is not likely to prevail because he did not have invitee status when he was climbing over the gate. In jurisdictions following the traditional landowner liability rules, the duty owed by an owner or occupier of land to those on the land depends on whether the person on the land is characterized as a trespasser, licensee, or invitee. A trespasser is one who comes onto the land without permission or privilege. A licensee is one who enters on the land with the landowner's permission, express or implied, for his own purpose or business rather than the landowner's benefit. An invitee is one who enters onto the premises in response to an express or implied invitation of the landowner. However, a person loses his status as an invitee if he exceeds the scope of the invitation-e.g., if he goes into a portion of the premises where his invitation cannot reasonably be said to extend. Here, the guest was an invitee while on the grounds of the resort, but he lost invitee status when he began climbing over the gate to get into the closed bar. He became a trespasser because he clearly did not have express or implied permission to climb into the bar, and a landowner owes no duty to an undiscovered trespasser. On the other hand, a landowner owes a discovered or anticipated trespasser the duty to warn of or make safe artificial conditions known to the landowner that involve a risk of death or serious bodily harm and that the trespasser is unlikely to discover. Here, while the guest can argue that he was an anticipated trespasser because others had climbed over the gate in the past, there is no evidence that the resort knew of the dangerous condition of the brackets, so the resort has breached no duty to the guest under these facts

On an icy day, a vehicle driven by the defendant struck the plaintiff's car in the rear, smashing a taillight and denting the plaintiff's bumper. Before the plaintiff could say anything, the defendant rushed out of his car and told the plaintiff, "Look, if you'll take $500 for the damage, I'm sure my insurance company will pay for it." The plaintiff refused and sued the defendant for damage to his car and minor personal injuries. The plaintiff wishes to testify as to the defendant's statement at the time of the accident. The defendant objects. Should the court allow the defendant's statement to be admitted? A.Yes, because it is a statement by an opposing party. B.Yes, because it is hearsay within the statement against interest exception. C. No, because the statement took the form of a settlement negotiation. D. No, because the statement is hearsay not within any exception

(A) The statement by the defendant, who is one of the parties to the action, is admissible as a statement by an opposing party (commonly called an admission). Federal Rule 801(d)(2) provides that a statement offered against a party that is the party's own statement is not hearsay and therefore cannot be excluded by the rule against hearsay. Assuming that it is relevant and not barred by other rules, the statement is admissible. Here, the defendant's statement is being offered against him at trial. It is relevant because it can be interpreted as a prior acknowledgment by the defendant that he was not totally blameless in the accident, which is undoubtedly inconsistent with his contentions at trial. The statement does not violate Federal Rule 408, which makes offers to compromise a disputed claim inadmissible to prove or disprove the validity or amount of a disputed claim, because it was made by the defendant before the plaintiff made any claim; i.e., there was not yet an actual dispute between the parties. Nor does the statement violate Rule 411, which bars evidence that a person has liability insurance when offered to show fault or ability to pay, because the defendant's reference to his insurance was an intrinsic part of his admission and could not be readily severed from it. No other rules barring relevant evidence apply, so the statement should be admitted.

A jogger found a stray dog in the park. She took the dog home with her and placed an ad in the paper to try to find the dog's owner. Soon thereafter, the owner of the dog contacted the jogger. He came to the jogger's home and identified the dog as his. He offered to pay the jogger a $200 reward at the end of the week. The jogger thanked the dog owner but turned down the reward. At the end of the week, however, the jogger changed her mind, so she called the dog owner and told him that she would like the reward after all. He refused to pay her, and she sues him for breach of contract. What will the jogger recover? A. Nothing, because she rejected the dog owner's offer. B. Nothing, because there was no consideration to support a contract. C. $200, because the technical defense of the Statute of Frauds will be overcome by the dog owner's moral obligation to pay. D. $200, because the dog owner could not have revoked his offer until the end of the week, and he failed to do so before the jogger accepted.

(B) The jogger will recover nothing because her finding the lost dog occurred prior to the dog owner's promise to pay the $200. An enforceable contract must be supported by consideration. Consideration consists of: (i) a bargained-for exchange between the parties; and (ii) an element of legal value to that which is bargained for. Legal value is present if the promisee has incurred a detriment (i.e., has done something she is under no legal obligation to do or has refrained from doing something that she has a legal right to do). To have a "bargained-for exchange," the promise must induce the detriment, and the detriment must induce the promise. If something has already been given or performed before the promise is made, it will not satisfy the bargain requirement, because it was not given in exchange for the promise. Here, the jogger was under no legal obligation to return the dog to its owner. Thus, in doing so, she incurred a detriment. However, the jogger was not induced to so act by the dog owner's promise to pay $200. Because the jogger's actions regarding the dog were performed before the dog owner's promise, those actions were not given in exchange for the promise when made. Thus, the "bargain" element is absent.

To combat fraud and misuse of driver's licenses, a state's department of motor vehicles enacted new regulations for the issuance of driver's licenses. One of the regulations, which were authorized by state law, required for the first time that driver's licenses display a photograph of the person whose name is on the license. The regulations did not provide for any exemptions from this requirement. Living entirely within the state was a religious sect whose followers devoutly believed that allowing oneself to be photographed was sinful. However, because much of the state was rural and sparsely populated, members of the sect needed to travel by automobile to obtain necessary services and to gather for worship. A member of the sect who was refused a driver's license because he would not allow himself to be photographed challenged the state regulation in federal court. Is the court likely to uphold the application of the regulation to the religious group? A. Yes, because exempting the church's members from the regulation would not have a secular purpose and would constitute improper state advancement of, and entanglement with, religion. B. Yes, because enactment of the regulation was not motivated by a desire to interfere with religion. C. No, unless the state shows that the regulation is necessary to promote a compelling governmental interest. D. No, because the opposition to the regulation arises from a sincerely held religious belief.

(B) The law will be upheld because it is a neutral law that is applicable to all drivers in the state. The Free Exercise Clause does not require exemptions from government regulations for a person whose religious beliefs prevent him from conforming his behavior to the requirements of the law. Unless the law was motivated by a desire to interfere with religion, it can be applied to regulate the conduct of one whose religious beliefs conflict with the law. Here, the sect member must allow his photograph to be taken if he wants to obtain a driver's license; the state is entitled to enforce this regulation because it is a neutral law of general applicability.

A landowner conveyed her 20-acre tract of land to a developer and his heirs, "provided that no multi-family dwellings may be built on the property for a period of 25 years. If such construction is undertaken, the grantor may terminate the conveyance and retake the land." Two years later the landowner died, leaving her nephew as the sole beneficiary under her will. Shortly thereafter, the nephew discovered that the developer was constructing multi-family dwellings on the land. He promptly brought an ejectment action against the developer. The jurisdiction in which the land is located has a statute providing that all future interests are freely devisable and alienable inter vivos. The common law Rule Against Perpetuities is unmodified by statute. To whom should the court rule that ownership of the land belongs? A. The nephew, because the developer began constructing multi-family dwellings on the land. B. The nephew, because the developer began constructing multi-family dwellings on the land and the nephew brought an action for ejectment. C. The developer, because the Rule Against Perpetuities applies. D. The developer, because the restriction in the conveyance is an invalid restraint on alienation.

(B) The nephew has ownership of the land because he has exercised his power to terminate the developer's fee simple subject to condition subsequent in the land. A fee simple subject to a condition subsequent is created when the grantor retains the power to terminate the estate of the grantee on the happening of a specified event. On the happening of that event, the estate of the grantee continues until the grantor exercises her power of termination (right of entry) by bringing suit or making reentry. Here, the landowner conveyed a fee simple subject to a restriction against constructing multi-family dwellings, retaining a right of entry. She devised this interest to the nephew in her will, as the statute permitted, and the nephew exercised this right by bringing an ejectment action against the developer after the condition was violated. Hence, the nephew will have ownership of the land.

During a nationwide trucker's strike, striking drivers committed repeated acts of violence against independent truckers and railroad shipments that had replaced truck transportation. This prompted Congress to enact an emergency measure directing the President to dispatch United States Army troops to specified cities and rail and highway locations to preserve order and ensure the continued flow of commerce. Is this enactment constitutional? A. No, because it infringes on the President's authority to faithfully execute the laws of the United States. B. No, because it infringes on the President's authority as Commander in Chief of the armed forces. C. Yes, under Congress's power to regulate commerce. D. Yes, under Congress's power to raise and support the armed forces.

(B) Although the President has no power to declare war, Article II, Section 2 makes the President commander in chief of the military, which affords the President extensive power to deploy military forces against any enemy, foreign or domestic. Congress lacks such power. Therefore, (B) is correct; this statute directly infringes upon the President's authority as commander in chief to make such orders as he deems proper with respect to the armed forces, and thus violates the doctrine of separation of powers. (A) is incorrect because the duty to execute the laws of the United States is an obligation, not a grant of authority. (C) is incorrect because even if the measure has some effect upon interstate commerce, it is still a violation of the separation of powers doctrine. Congress's power under the Commerce Clause does not supersede other powers that the Constitution has specifically bestowed on another branch of government. (D) is incorrect because the enactment does not appropriate money to support the armed forces, but seeks to control their activities

A car owner sued a crane operator for negligence in a federal court because the crane operator backed over and crushed her car. During jury selection, the car owner's lawyer wished to exercise one of her peremptory challenges to excuse some potential jurors. Which of the following reasons would provide the best grounds for the crane operator's lawyer to object to the peremptory challenge? A. Excusing the only potential male juror because he is a crane operator, just like the defendant. B. Excusing a second African-American juror. C. Excusing the juror with multiple tattoos. D. Excusing the juror who has been unemployed for two years.

(B) Peremptory challenges allow an attorney to disqualify a potential juror because the juror displays an attitude or some characteristic that appears unfavorable to the attorney's client but that does not rise to the level of bias that would present grounds for a challenge for cause. A party cannot use peremptory challenges if the court suspects the challenge is for race, national origin, religion or gender, which violate the juror's equal protection rights under the Fourteenth Amendment. If the court detects a pattern of excluding jurors for any of these reasons, the opposing party can object, and the party may be required to justify the challenge by providing other nonobjectionable means. Here, exercising a peremptory challenge to excuse a second African-American juror from the panel without any other reason appears that it may be based on race. Therefore, choice (B) provides the best choice for the defense to object to the plaintiff's peremptory challenge.

A county initiated a tax foreclosure action against a landowner for delinquent taxes. The landowner executed and delivered to the county a quitclaim deed to his land. The county recorded its deed. Due to a clerical error, the land was not removed from the foreclosure action. The following month, the land was sold at the foreclosure sale and a sheriff's deed was delivered to the buyer. The buyer then sold the land to a friend and delivered to her a general warranty deed. The county initiated a quiet title action and obtained a judgment in its favor, and then began ejectment proceedings against the friend. If the friend brings an appropriate action for damages against the buyer, for whom should the court rule? A. The friend, because the buyer's title was unmarketable. B. The friend, because the covenants of quiet enjoyment and warranty were breached. C. The buyer, because he had title to the land at the time he conveyed it to the friend. D. The buyer, because a sheriff's deed has priority over a quitclaim deed.

(B) The court should rule for the friend because the buyer is liable on these covenants. The covenants of quiet enjoyment and warranty, which are contained in a general warranty deed, are breached when a third party lawfully interferes with the possession of the grantee or her successors. Here, the buyer is liable because he conveyed the land to the friend with a general warranty deed and the county is bringing lawful ejectment proceedings against the friend. (A) is incorrect because the time to object regarding marketability of title has passed. There is an implied covenant in every land sale contract that at closing the seller will provide the buyer with marketable title, i.e., title that a reasonably prudent buyer would accept in the exercise of ordinary prudence. A claim that someone other than the seller holds paramount title to the land would render title unmarketable. However, at closing the contract merges with the deed, so that the seller is no longer liable on the implied contractual covenant of marketable title.

A debtor owed a creditor $5,000, but the debt was barred by the applicable statute of limitations. The debtor agreed to assign to the creditor a $4,000 debt that was owed to him by a third party and was coming due in a week. The debtor called the third party to inform him of the assignment. When the debt became due, the third party refused to pay the creditor. The creditor brings an action to collect the debt against the third party. Will the creditor likely prevail? A. Yes, because the creditor's agreement to accept a lesser amount than the original debt constituted consideration for the assignment. B. Yes, because an assignment need not be in writing to be enforceable. C. No, because the third party may raise the debtor's statute of limitations defense on the original debt. D. No, because a new promise to pay a legal obligation barred by law must be in writing.

(B) The creditor will prevail because the debtor has made an effective assignment of his right to collect the debt from the third party. The general rule is that a writing is not required to have an effective assignment. Here, the oral assignment to the creditor of the debtor's right to the $4,000 was effective and enforceable by the creditor. (A) is incorrect for two reasons: The creditor's agreement to accept $4,000 did not constitute consideration because he no longer had a right to enforce the original debt. Furthermore, consideration is not required for an assignment; a gratuitous assignment is effective. The absence of a writing or consideration may allow the assignor (the debtor) to revoke an assignment, but will not prevent the assignee from enforcing it against the obligor.

A strawberry farmer held his farm open to the public to pick strawberries for a fee. The farmer knew that many patrons would eat as many strawberries out in the field as they would bring home with them, so he advertised that no chemical pesticides or fertilizers were used on his strawberries. The owner of the land adjacent to the farm began operating a soap factory, a use allowed by the zoning code. Flakes of an unavoidable chemical byproduct of the soap-making process would drift over onto the farm whenever the wind was blowing in that direction and settle onto the strawberry plants. The flakes caused no harm to the plants themselves but detracted from the appearance of the strawberries as well as their taste if eaten right off the plant; consequently, the farmer's business declined. On several occasions, the farmer complained to the factory owner, but the owner did nothing, in part because a visit to the county recorder of deeds office had convinced him that he was the true owner of a large part of the strawberry farm, although in fact it was just a recording error. Can the farmer recover damages for the harm caused to his business from the factory owner? A. Yes, because the discharge from the owner's factory entered the farmer's land. B. Yes, because the factory owner intended to conduct the activities that caused the particles to fall on the farmer's land. C. No, because the factory owner had no intent to cause harm to the farmer's property. D. No, because the factory owner's belief that he owned the property, although erroneous, was reasonable.

(B) The farmer can recover damages for trespass to land because the factory owner intended to conduct the activities that caused the trespass. To establish a prima facie case for trespass to land, plaintiff must prove: (i) an act of physical invasion of plaintiff's real property by defendant; (ii) intent on defendant's part to bring about the physical invasion; and (iii) causation. Here, flakes of the chemical byproduct of the factory owner's soap factory physically invaded the farmer's property when the wind blew. The factory owner intended to bring about the trespass because, after the farmer had complained, the factory owner knew with substantial certainty that the flakes would continue to fall on the farm whenever the wind was right. Finally, the factory owner's operation of the soap factory was the cause of the flakes settling on the farmer's strawberries, completing the prima facie case of trespass to land.

A daughter owed her father $1,250. The father's best friend was having financial difficulties and the father wanted to help him, so the father told his daughter to pay the $1,250 to his friend when the debt came due in three days. Immediately after directing his daughter to pay his friend, the father called his friend and told him he should expect to get $1,250 from his daughter in three days. When the debt came due, the daughter tendered the $1,250 to her father instead of to his friend, and the father accepted the money. The friend sues the daughter for $1,250. Which of the following is the most likely result? A. The friend will recover, because the father effectively assigned his right to collect the $1,250 to his friend. B. The friend will not recover, because the father's acceptance of $1,250 from his daughter revoked his gift to his friend. C. The friend will not recover, because the daughter was never indebted to him and cannot be forced to pay him. D. The friend will not recover, because the daughter's tender of $1,250 to her father, and her father's acceptance of the money, constituted a novation.

(B) The father validly assigned his right to receive the money to his friend. However, this assignment was revocable, and it was revoked when the father accepted the money from his daughter. The father's right to receive the money from his daughter was a right that could be assigned. By telling her to pay the money to his friend, the father manifested an intent to transfer his rights completely and immediately to his friend. Neither a writing nor consideration was required for this assignment to be valid. However, these factors do affect revocability. This assignment was not given for value. Such a gratuitous assignment is generally revocable. Among the exceptions to this rule are situations where the obligor has already performed, or where the assignor is estopped from revoking by virtue of the fact that he should reasonably foresee that the assignee will change his position in reliance on the assignment and such detrimental reliance occurs. Here, the daughter (the obligor) has not already performed the terms of the assignment. On the contrary, she tendered performance directly to the original obligee. Also, there is no indication that the friend in fact changed his position detrimentally in reliance on the assignment. Consequently, the general rule of revocability of a gratuitous assignment applies. One way in which a gratuitous revocable assignment may be terminated is by the assignor taking performance directly from the obligor. By accepting the money from his daughter, the father (the assignor) took direct performance from the obligor, thereby revoking the assignment. As a result, his friend has no right to the money.

An owner purchased a home in a new subdivision, paying 20% of the purchase price as a down payment and financing the rest of her purchase through a mortgage with a lender. The owner lived in her home for three years and always made her mortgage payments promptly. She then decided to put her house on the market. While the house was being marketed, the owner continued to make all mortgage payments promptly. She sold the house to a buyer, who purchased the property subject to the mortgage. After the buyer took possession, the lender received no further mortgage payments from either the owner or the buyer. In most states, which of the following best describes the remedy or remedies available to the lender? A. The lender may foreclose on the land, but may not sue either the owner or the buyer on the underlying debt. B. The lender may foreclose on the land or it may sue the owner on the underlying debt. C. The lender may foreclose on the land or it may sue the buyer on the underlying debt. D. The lender may foreclose on the land or it may elect to sue either the owner or the buyer on the underlying debt.

(B) The lender may foreclose on the land or it may sue the owner on the underlying debt. When a mortgagor sells the mortgaged property and conveys a deed, the grantee takes subject to the mortgage, which remains on the land. However, the mortgagor remains primarily and personally liable on the loan (unless the grantee has signed an assumption agreement). In most jurisdictions, when a sale is made "subject to" the mortgage, the mortgagee has the option of foreclosing on the land or suing the mortgagor (in this case, the owner) on the debt. (A) is incorrect because it fails to allow for the remedy of suing on the debt

An American tourist was visiting another country when he was warned by United States health authorities to go immediately to a hospital because he had a serious and extremely contagious disease that required him to be quarantined. He decided to ignore the warning and instead traveled on an airline flight back to the United States. Despite the tourist's belief that he would not be discovered and his best efforts to keep a low profile, the news media were tipped off to what he had done and publicized it. When a passenger who had been sitting next to the tourist on the plane learned about it, she became extremely upset, fearing that she would contract the disease. The passenger brought a negligence action to recover for the distress she suffered but the jury rejected her claim. Why? A. The tourist's conduct was not extreme and outrageous. B. The passenger did not suffer physical injury from her distress. C. The passenger did not contract the disease from the tourist. D. The tourist could not have reasonably foreseen that the other passengers would find out about what he had done.

(B) The passenger did not prevail because she did not suffer physical injury from her distress. The duty to avoid negligent infliction of emotional distress may be breached when the defendant creates a foreseeable risk of physical injury to the plaintiff. Typically in that type of situation, the emotional distress must cause some physical symptoms. In most jurisdictions, emotional distress without physical symptoms is insufficient when the plaintiff's distress was caused by being within the zone of danger from the defendant's negligent conduct. Here, the tourist created a risk of physical injury (the contagious disease) to those around him, such as the passenger. She suffered distress as a result of the threat, but her distress did not cause physical injury. While recovery has been allowed without proof of physical symptoms in situations where the defendant's negligence was directed at a close family member or is based on a special relationship between the plaintiff and the defendant, those situations are inapplicable here. (A) is incorrect because the tort of negligent infliction of emotional distress does not require that the defendant's conduct be extreme and outrageous. That is an element of intentional infliction of emotional distress, which also requires that the defendant intend that the plaintiff suffer severe emotional distress, or act in reckless disregard of a high probability that emotional distress will result. Here, the tourist had no such intent, nor is there evidence that he believed there to be a high probability that anyone would suffer emotional distress, given his efforts to keep his unauthorized travel quiet. (

On August 5, the owner of a hot dog plant and the proprietor of a local ballpark concession stand entered into a written agreement providing, among other things, that if the local team wins the state championship, the plant owner will deliver to the proprietor 500 hot dogs on each of the following days: September 5, 7, and 9. The price was set at 25 cents per hot dog, with payment to be made on September 10 by the proprietor to a creditor of the plant owner. On August 15, the plant owner decided that he wanted to avoid his obligation to deliver the hot dogs. The creditor has not become aware of the agreement between the plant owner and the proprietor. Which of the following is the most accurate statement? A. The plant owner cannot rescind the contract without the permission of both the proprietor and the creditor. B. The plant owner cannot rescind the contract without the permission of the proprietor. C. The plant owner can repudiate the agreement because the promise to perform by the proprietor is illusory. D. The plant owner can revoke the offer to sell hot dogs if the team does not win the state championship.

(B) The proprietor must agree to a rescission of the agreement. A party to a contract may not unilaterally rescind it if the contract is valid (i.e., in the absence of mistake, misrepresentation, etc.). However, both parties to a contract may agree to rescind and discharge their contractual duties as long as the duties are still executory on both sides. Here, neither party has performed under the contract, so the contract will be mutually rescinded if the proprietor gives his assent to the plant owner.

A seller entered into an enforceable written agreement to sell her house to a buyer for $425,000. The agreement provided that closing would take place on September 18, and on that date the seller would provide marketable title, free and clear of all encumbrances. The agreement was silent as to risk of loss if the house was damaged prior to closing and as to any duty to carry insurance. On August 31, the seller cancelled her homeowners' insurance when she moved out of the house. Consequently, when the house was destroyed by wildfires on September 15, it was uninsured. The buyer refused to close on September 18 and the seller immediately brought an action against him for specific performance. The buyer countersued for the cancellation of the contract and return of his earnest money. Both parties stipulate that the value of the property without the house is $225,000. In this jurisdiction, which has no applicable statute, is the seller likely to prevail? A. Yes, but the price will be abated to $225,000. B. Yes, for the full contract price. C. No, because the seller had a duty to carry insurance until the closing date. D. No, because the seller could not convey marketable title.

(B) The seller will most likely prevail for the full contract price. Although jurisdictions differ as to which party has the risk of loss, the majority rule is that where property subject to an enforceable contract for sale is destroyed without the fault of either party before the date set for closing, the risk of loss is on the buyer. Thus, the buyer must pay the contract price despite a loss due to fire, unless the contract provides otherwise. Here, the house was destroyed by fire after the seller and buyer entered into their contract for the sale of the house, but before the closing date. The contract was silent regarding the risk of loss. Thus, under the majority rule, the risk of loss is on the buyer. As a result, the seller is entitled to receive specific performance of the contract, meaning that the buyer must pay the full contract price

A landowner who had owned and operated a small airport notified the electric company that he was discontinuing operations and that it should shut down the electrical current that had supplied his communications equipment. The equipment had been surrounded by a fence and signs warning of high voltage. Because the electric company had maintained a transformer next to the landowner's communications equipment that contained many valuable and reusable parts, it decided to leave the power on to prevent theft until it could schedule removal of the transformer. Three days later, a trespasser who knew that the airport had closed went onto the property looking for something to steal. He could find nothing of value except the transformer. He noticed the signs warning of the high voltage but believed that the power had since been turned off. He scaled the fence with the intent to dismantle the transformer. As soon as he touched the transformer, he was seriously injured by the electric current. If the trespasser asserts a claim against the electric company to recover damages for his injuries, will he prevail? A. Yes, because the electric company was not the owner of the land on which the trespasser trespassed. B. Yes, because the electric company used unreasonable force to protect its property. C. No, because the trespasser was a trespasser on the landowner's land. D. No, because the trespasser intended to steal the electric company's transformer.

(B) The trespasser will prevail against the electric company because it did not have the right to use deadly force to protect its property. As a general rule, one may use reasonable force to prevent the commission of a tort against one's property. However, force that is likely to cause death or serious bodily harm is not permitted when the invasion is threatening property alone. Furthermore, one may not use indirect deadly force when such force could not lawfully be directly used. Because the trespasser was threatening only the property interest of the electric company, the use of deadly force would not be privileged against him. By leaving the power on to prevent theft, the electric company was using indirect deadly force to defend its property where such force could not lawfully be directly used. Hence, it will be liable to the trespasser for his injuries. (A) is incorrect because the electric company's status with respect to the land is irrelevant for the type of claim that the trespasser is asserting. In negligence actions, the limited duty to trespassers that a landowner has is not shared by persons with an easement or license to use the land; they owe a duty of reasonable care even to trespassers. Here, however, the theory of the trespasser's claim is not negligence but more likely battery, because the electric company intended to leave the power on to protect its property and will be deemed to have intended the consequences of that conduct. Even if the electric company had been the landowner, it would not have been privileged to leave the power on solely to protect its property from theft.

As permitted by state law, a large city in the state adopted an ordinance legalizing slot machines in shopping malls within the city. Several prominent city residents were upset by the new ordinance because gambling violates one of the main tenets of their religion. Seeking relief, the citizens contacted their representative in Congress and asked the representative to sponsor a bill making it illegal to place gambling machines in shopping malls throughout the country. The representative sponsored such a bill. Congress made a factual finding that the activity regulated has a substantial economic effect on interstate commerce and passed the statute. If the statute banning gambling machines in shopping malls is challenged on constitutional grounds by a proper plaintiff in federal court, would the court likely uphold the statute? A. No, because it was based on the citizens' religious tenets and so violates the First Amendment Establishment Clause. B. No, because the statute does not regulate the channels or instrumentalities of interstate commerce. C. Yes, because Congress has made a factual finding that the activity regulated has a substantial economic effect on interstate commerce. D. Yes, because there is a conceivable rational basis for concluding that the activity regulated, in aggregate, substantially affects interstate commerce.

(C) The court would likely uphold the statute against a constitutional challenge because there is a conceivable rational basis for concluding that gambling, in aggregate, substantially affects interstate commerce. Under the Commerce Clause, Congress may regulate: (i) the channels of interstate commerce; (ii) the instrumentalities of interstate commerce, as well as persons and things in interstate commerce; or (iii) activities that have a substantial effect on interstate commerce. When Congress attempts to regulate intrastate activities under the third of the preceding prongs, the Court will uphold the regulation if it involves economic or commercial activity as long as there is a conceivable basis for concluding that the activity in aggregate substantially affects interstate commerce. Here, the statute involves commercial activities-placing gambling machines in shopping malls. And it is conceivable that gambling machines in shopping malls, in aggregate, could substantially affect interstate commerce. Therefore, the statute would be upheld. (C) is incorrect because it states the test applicable to situations where Congress seeks to regulate noneconomic or noncommercial intrastate activity under the Commerce Clause (e.g., possession of a gun in a school zone); as noted above, gambling in shopping malls is an economic or commercial activity.

Federal legislation provided that the marketing and sale of oranges was subject to the control of a local marketing authority. The marketing authority determined what quantity of oranges could be sold by each grower, the price, and the location of sale. These decisions were made by a council of local growers whose members were selected by the federal Department of Agriculture. The applicable federal legislation provided, in part, that when any grower subject to a marketing order challenged the propriety of that order, the council of the marketing authority must submit the controversy to the United States district court with geographical jurisdiction for a recommendation as to whether the order should be confirmed, modified, or rescinded. After the hearing in district court, the council must revote on the challenged marketing order. A citrus grower brings suit in United States district court, seeking on constitutional grounds to enjoin enforcement of the federal legislation providing for the marketing order that the council issued with regard to his orange crop. If the court rules on the constitutional issue, is the grower likely to prevail? A. No, because the federal government may properly regulate items in interstate commerce. B. No, because the marketing order system is a necessary and proper means of effectuating the commerce power. C. Yes, because the federal legislation permits the federal district court to give an advisory opinion. D. Yes, because the federal legislation deprives the grower of his property without due process of law.

(C) Because the federal legislation merely allows the district court to issue a "recommendation," the legislation permits the rendition of advisory opinions. Article III of the United States Constitution establishes the basis for the judicial power of federal courts. It provides that the judicial power extends to "cases and controversies." Although Congress has power to delineate the jurisdictional limits of Article III courts, it is bound by the standards of judicial power set forth in Article III as to subject matter, parties, and the requirement of "case or controversy." Thus, Congress cannot require these courts to render advisory opinions or perform administrative or nonjudicial functions. The federal orange marketing legislation at issue here does not give federal district courts the authority to render binding decisions in final resolution of a controversy. Rather, the legislation simply allows the courts to make a recommendation as to confirmation, modification, or rescission of a challenged marketing order. This recommendation is apparently nonbinding on the parties and is followed by a re-vote of the marketing council. These circumstances indicate that, under this federal legislation, an Article III federal court would be rendering an advisory opinion in violation of the Constitution.

A backpacker came upon another hiker who had been bitten by a rattlesnake. The backpacker carried the bitten hiker back to his vehicle and drove him toward the nearest hospital. On the way there, while exceeding the posted speed limit, the backpacker lost control of his vehicle and crashed into a tree by the side of the road. He was uninjured, but the snakebitten hiker's leg was broken. An ambulance soon arrived and took the hiker to the hospital. The emergency room physician committed malpractice that resulted in the loss of the hiker's leg. The hiker is now suing the backpacker. Which of the following is the most likely reason why the backpacker will be held liable for the hiker's injuries? A. Having undertaken to rescue the hiker, the backpacker is strictly liable for injuries resulting from the rescue. B. The emergency room physician's malpractice is a foreseeable intervening cause that does not relieve the backpacker of liability. C. The backpacker did not conduct himself as a reasonably prudent person in carrying out the rescue of the hiker. D. The backpacker committed negligence per se when he exceeded the posted speed limit.

(C) If the backpacker undertakes to rescue the hiker, he must be reasonably prudent in doing so. The general rule in tort law is that no legal duty is imposed on any person to affirmatively act for the benefit of others. However, one who gratuitously acts for the benefit of another is then under a duty to act reasonably. If he acts negligently, he will be liable for damage caused thereby. (A) is an incorrect statement of the law-a rescuer is not strictly liable for a victim's injuries, but rather is liable only for negligent acts. (B) is an accurate statement of the law but does not take into account that the backpacker must be negligent to be liable at all. (D) is incorrect because it is not necessarily true. A violation of a statute will not be negligence per se where compliance would cause greater risk of harm than violation, such as in an emergency. If it was necessary to speed to get the hiker to the hospital for treatment of the snakebite, it may have been excusable to exceed the posted speed limit. If the backpacker was not otherwise negligent, this would not establish negligence at all.

A plaintiff filed a negligence action against a defendant in federal district court, seeking damages for personal injuries suffered in a traffic accident. The plaintiff timely served on the defendant a request for production of certain documents. The defendant objected to part of the request, claiming that it sought information beyond the scope of discovery. The defendant honestly believed this to be true, but case law clearly held to the contrary. After conferring with the defendant and not resolving the matter, the plaintiff filed a motion to compel the defendant to produce the documents. If the court orders the defendant to produce the documents, what sanctions may the court impose on the defendant at that time? A. The court may enter a default judgment against the defendant. B. The court may strike relevant pleadings of the defendant or enter an order precluding the defendant from offering evidence on certain claims or defenses. C. The court may not impose sanctions at this time, but it must require the defendant to pay the reasonable expenses incurred by the plaintiff in making the motion to compel. D. The court at this time may neither impose sanctions nor require the defendant to pay any expenses incurred by the plaintiff.

(C) The court may not impose sanctions on the defendant, but it must require the defendant to pay the plaintiff's reasonable expenses incurred in making the motion. If a motion to compel is granted, the court must require the opposing party to pay the movant's reasonable expenses incurred in making the motion. However, the court may not order this payment if the movant filed the motion before attempting to seek production without a court order, if the nondisclosure was substantially justified, or if other circumstances exist that make an award of expenses unjust. [Fed. R. Civ. P. 37(a)(5)(A)] None of these exceptions appear applicable here.

A homeowner discovered that the siding on his house was defective and had allowed water to enter the structure, causing damage to the wood framing. The homeowner tried for some time to negotiate a settlement with the corporation that the homeowner believed had manufactured the defective siding. When no settlement was forthcoming, the homeowner filed an action in federal district court against the corporation one week before the statute of limitations expired. Service of process was effected on the corporation several months later. After inspecting the home, the corporation filed and served its answer in which it denied manufacturing the siding used on the homeowner's house. Upon examining the corporation's evidence, the homeowner conceded that the siding was manufactured by another company. With leave of the court, the homeowner then filed an amended complaint substituting the actual manufacturer of the siding for the original incorrect defendant. The amended complaint was served on the manufacturer approximately seven months after the original complaint was filed and after the statute of limitations had expired. The manufacturer was unaware of the action until it was served with the amended complaint. The manufacturer filed a motion for summary judgment on the grounds that the homeowner's claim against it is barred by the statute of limitations. How should the court rule on the motion? A. Deny the motion, because the homeowner is entitled to recover from the actual manufacturer the damages caused by the defective siding. B. Deny the motion, because the amended complaint relates back to the time the original complaint was filed, which was before the statute of limitations expired. C. Grant the motion, because the amended complaint was filed after the statute of limitations expired and the actual manufacturer did not receive timely notice of the action. D. Grant the motion, because amendments to pleadings may not be used to substitute an entirely new defendant unrelated to the one originally named.

(C) The court should grant the motion for summary judgment on the basis that the claim is barred by the statute of limitations. Amendments substituting a new defendant for one originally named are allowed and the amendment will relate back to the time the original complaint was filed if (i) the claims in the amendment arise from the same transaction or occurrence as the claims set out in the original pleading; and (ii) within the time allotted for serving the original complaint (90 days from filing per Federal Rule 4(m)), the new defendant received such notice of the action that it will not be prejudiced in defending on the merits and knew or should have known that the action would have been against it (the new defendant). Here, the manufacturer received no notice of the action within the time allotted for service and could not have known about the plaintiff's mistake in naming the defendant. The amendment thus does not relate back and is time-barred. (A) is incorrect because it ignores the statute of limitations issue. (B) is incorrect because an amendment changing the party only relates back if it meets the requirements listed above. (D) is incorrect because a plaintiff may use an amendment to change a party or naming of a party.

At the defendant's trial for grand theft auto and other offenses, the prosecution offers to introduce the testimony of a police officer. The officer will testify that he showed a photographic lineup containing the defendant's picture to a witness who had seen the defendant fleeing from the stolen vehicle at the conclusion of a high-speed chase, and the witness selected the defendant's picture. The witness has left the state and she refuses to return. Should the court admit the evidence? A. Yes, because the witness is unavailable to testify. B. Yes, because it is a prior identification. C. No, because it is inadmissible hearsay. D. No, because the picture has not been properly authenticated.

(C) The court should not admit the evidence because it is hearsay not within any exception. The officer is attempting to testify to the statement of a witness (an out-of-court declarant) that the defendant was the person she saw fleeing from a stolen vehicle. The testimony is being offered to prove the truth of the matter asserted-that the witness saw the defendant flee from a stolen vehicle. This constitutes inadmissible hearsay because it does not fit within any exception to the hearsay rule and should not be admitted.

During the investigation of a large gambling operation, the police obtained a warrant to search a bookie's home based on the affidavit of an informant. The informant was a rival bookie who had never acted as an informant before, and much of the substance of the rival's information came from third-party sources. During the search, the police seized a variety of gambling evidence, including betting slips and a check from the defendant. The bookie and the defendant were arrested for violating the state's gambling laws, and separate trials were ordered. At a suppression hearing for the bookie, the court held that the search warrant for the bookie's home was not supported by probable cause and suppressed introduction of the evidence seized. The defendant moved to suppress introduction of the betting slips and the check on the same basis. If the court agrees that the search warrant of the bookie's home was not supported by probable cause, should the defendant's motion be granted? A. Yes, because the rival bookie was not a reliable informant.' B. Yes, because the evidence is the fruit of an unlawful search. C. No, because the defendant's reasonable expectation of privacy was not constitutionally violated. D. No, because the police acted reasonably in relying on the issuance of the warrant.

(C) The defendant's motion should be denied because his constitutional rights were not violated by the search and seizure of the bookie's home based on an invalid warrant. To have a Fourth Amendment right to be free from unreasonable search and seizure, a person must have a reasonable expectation of privacy in the place searched or the item seized. Standing to challenge a search on Fourth Amendment grounds does not exist merely because a person will be harmed by introduction of evidence seized during an illegal search of a third person's property; the defendant's own expectation of privacy must be violated. Here, the defendant had no right of possession of the place searched and no property interest in the items seized; thus, he had no standing to object to the search of the bookie's home and the seizure of the betting slips and check

In a civil action tried to a jury, the defendant objected to the introduction by the plaintiff of certain evidence without the judge's first making a preliminary ruling on the admissibility of the evidence. For which evidence is the defendant's objection NOT appropriate? A. Opinion testimony regarding the structural integrity of a building by an engineer called by the plaintiff, without a preliminary determination by the judge that the engineer is an expert. B. Hospital records pertaining to the plaintiff offered by the plaintiff, without a preliminary determination by the judge that they were made as a regular activity of the hospital staff. C. Contract negotiations between the plaintiff and a third party, without a preliminary determination by the judge that the third party was the defendant's agent. D. A paramedic's testimony that the plaintiff's wife, before she died, said that the defendant's car went through a red light before hitting her, without a preliminary determination by the judge that she made the statement under a sense of impending death.

(C) The defendant's objection to the contract negotiations is not appropriate because whether an agency relationship existed is determined by the jury. The Federal Rules of Evidence distinguish preliminary facts to be decided by the jury, which determine whether the offered evidence is relevant to the issues in the case, from preliminary facts to be decided by the judge, which determines whether the offered evidence is competent to be admitted at all. Whether an agency relationship existed between the defendant and a third party is a question of fact to be decided by the jury; if the jury decides that the third party was not the defendant's agent, it will disregard as irrelevant the evidence of contract negotiations undertaken by the third party. While the judge must find that the proponent of the contract negotiations has introduced enough evidence to allow the jury to find that an agency relationship existed, the ultimate determination of agency rests with the jury. (A) is incorrect because the judge must determine the qualifications of a witness called as an expert before permitting the witness to offer an opinion or conclusion on a matter appropriate for expert testimony. If the judge decides that the engineer does not qualify as an expert, he will not be permitted to testify on the structural integrity of the building. (B) and (D) are incorrect because all preliminary fact questions involving the standards of trustworthiness of exceptions to the hearsay rule must be determined by the court. Thus, the court must decide whether a purported business record was made in the regular course of business, and whether a statement offered as a dying declaration was made under a sense of impending death.

A lawyer rented an office building for his law practice and subleased most of the building to three other tenants. The lawyer paid $2,000 per month to the owner and charged his subtenants $600 per month each. After having been in the building for three years, the lawyer and the owner orally agreed that the lawyer would purchase it for a price of $120,000, to be paid in monthly installments of $2,000 over a five-year period. It was further agreed that title would remain in the owner's name until $48,000 had been paid on the total price, whereupon the owner would deliver a deed to the lawyer. Shortly thereafter, the lawyer spent $4,000 redecorating his suite. During the course of the next two years, the lawyer hired an associate and placed her in one of the offices formerly occupied by one of the subtenants, and raised the monthly rental he charged the other two subtenants to $700. Two years after the agreement with the owner, the lawyer demanded that the owner convey the building by delivery of a deed. The owner refused, denying that any oral agreement for sale had ever existed. The lawyer brings an action for specific performance against the owner, who pleads the Statute of Frauds as a defense. If the owner wins, what is the likely reason? A. The lawyer did not obtain the owner's approval before making the improvements to his offices. B. The original violation of the Statute of Frauds was incurable. C. The lawyer's actions in paying $2,000 per month and making improvements were as consistent with being a tenant as with the oral contract. D. The owner received no unconscionable benefit entitling the lawyer to equitable relief.

(C) The doctrine of part performance may be used to enforce an otherwise unenforceable oral contract of sale, provided the acts of part performance unequivocally prove the existence of the contract. Here, the lawyer's actions are explicable even if he had remained a tenant, because he continued to pay the same amount per month as had been previously paid as rent, and the improvements he made are a kind frequently made by long-term tenants. (A) is incorrect because the owner's approval was not needed to make such improvements. (B) is incorrect because if the lawyer had taken actions that clearly indicated the presence of an oral contract, the doctrine of part performance would have applied. The benefit to the owner is irrelevant for purposes of the Statute of Frauds; the issue is part performance.

An employee who was fired plans to sue her former employer, claiming that the employer is liable for both wrongful termination under state law as well as violation of federal employment discrimination statutes, claiming total damages in the amount of $50,000. The employer and employee are citizens of the same state. May the employee assert these claims together in federal district court? A. No, because the amount in controversy does not exceed $75,000. B. No, because the federal court lacks subject matter jurisdiction over a state law claim between two citizens of the same state. C. Yes, because the federal court has federal question jurisdiction over . the federal statutory claim and supplemental jurisdiction over the state law claim. D. Yes, because federal courts have subject matter jurisdiction over all claims joined with federal statutory claims.

(C) The employee may assert these claims together. Federal question jurisdiction is available when the plaintiff, in his well-pleaded complaint, alleges a claim that arises under federal law. Here, the federal employment discrimination claim arises under federal law. Thus, a federal court has federal question jurisdiction over the claim. When the federal court has subject matter jurisdiction over one claim, it has discretion to exercise supplemental jurisdiction over related claims that derive from the same common nucleus of fact and are such that a plaintiff would ordinarily be expected to try them in a single judicial proceeding. (Essentially, this means that the supplemental claim must arise from the same transaction or occurrence as the claim invoking federal subject matter jurisdiction.) In the instant case, the federal employment discrimination claim is very closely related to the state wrongful termination claim. Each will deal with much of the same factual issues, and it would be expected for a plaintiff to bring such claims in a single action if at all possible. Thus, the court has supplemental jurisdiction over the state wrongful termination claims.

A builder entered into a contract with a landowner to build a warehouse for $500,000 by August 1. The agreement provided for five progress payments of $100,000 each at various stages of completion. On June 20, after the builder had spent $350,000 on performance and received $300,000 in progress payments, the builder notified the landowner that he was quitting the project. The landowner hired another contractor to complete the warehouse by August 1 for $250,000, which was a reasonable price given the short deadline. Which of the following statements regarding the parties' remedies is correct? A. The builder can recover $50,000, the difference between the amount he expended on performance and the amount he was paid, to prevent the landowner's unjust enrichment. B. Neither party can recover anything, because the $50,000 extra that the landowner had to pay to complete the building is offset by the $50,000 difference between the builder's expenditures and the payments the landowner made to him. C. The landowner can recover $50,000, the difference between the contract price and the total amount he paid for completing the building. D. The landowner can recover $100,000, the difference between the contract price and the total amount spent constructing the building.

(C) The landowner can recover $50,000, which is the amount above the contract price that it will cost to get the building completed. In construction contracts, the standard measure of damages when the builder breaches will depend on when the breach occurred. If the builder breaches after partially performing, the owner is entitled to the cost of completion plus reasonable compensation for any delay in performance (unless completion would involve undue economic waste). Here, the cost of completion (the amount above the contract price that it will cost to get the building completed) is $50,000, which was a reasonable price considering the deadline. Hence, that is what the landowner can recover. Most courts will allow the builder to offset or recover for work performed to date if necessary to avoid the unjust enrichment of the owner.

An applesauce bottler wishing to redesign his factory entered into a written contract with a contractor. The contract provided that the contractor would design and install a new glass bottle system, replacing the plastic bottle system, by March 10, and the bottler would pay the contractor a total of $100,000. The contract provided for a first payment of $50,000 on completion of the design plans and a second payment of the balance after installation and successful testing of the system. The contractor presented the bottler with the finished design plan in January, and the bottler paid him $50,000. The contractor ran into difficulty procuring the parts called for by his design, and this delayed the installation. The installation and testing were completed on March 25. Disappointed by the delay, the bottler now refuses to pay the contractor anything further. If the contractor sues the bottler, which party is likely to prevail? A. The bottler, because installation by March 10 was an express condition of the agreement. B. The bottler, because the doctrine of substantial performance does not apply to commercial contracts. C. The contractor, because the contract did not provide that time was of the essence. D. The contractor, because the agreement did not contain a liquidated damage clause for delay in completion.

(C) The late installation does not justify the refusal to pay. Unless the nature of the contract is such as to make performance on the exact day agreed upon of vital importance (e.g., contract for use of a wedding chapel), or the contract by its terms provides that time is of the essence, failure by a promisor to perform at the stated time will not be material. Merely providing a date for performance does not make time of the essence. Nothing in the contract here states that time is of the essence, and the contract does not by its nature require timely performance. This is a services contract, rather than a goods contract governed by the UCC, so perfect performance is not required. Thus, late performance is treated as a minor breach that gives the nonbreaching party a right to damages but does not relieve him of his duty to perform. The bottler still has the duty to pay the $50,000 but would be entitled to an offset for the damages suffered due to the delay.

A mob enforcer shot a pawnshop owner in the kneecap, intending to put him in the hospital because he was not paying his protection fees to the mob. However, the pawnshop owner hit his head on the edge of the counter when he fell. He suffered a blood clot and died as a result. A statute in the jurisdiction provides that a criminal homicide constitutes murder in the first degree when it is committed by an intentional and premeditated killing, murder in the second degree when it is committed while the defendant is engaged in the commission of a dangerous felony, and murder in the third degree for all other types of murder at common law. Another statute provides that manslaughter is a killing in the heat of passion on adequate legal provocation or a killing caused by criminal negligence. The crimes below are listed in descending order of seriousness. If the enforcer is charged with the pawnshop owner's killing, what is the most serious crime for which he can be convicted? A. Murder in the first degree, because the killing was the result of intentional and premeditated conduct. B. Murder in the second degree, because the killing occurred during the commission of the felony of assault with a deadly weapon. C. Murder in the third degree, because the enforcer had the intent to commit serious bodily harm. D. Manslaughter, because the enforcer acted with criminal negligence.

(C) The most serious crime for which the enforcer can be convicted is murder in the third degree. Under the statute in the question, all murders are third degree murder unless the prosecution proves any of the stated requirements for first or second degree murder, and here the facts do not establish any of those requirements. (A) is incorrect. The enforcer did not commit an intentional and premeditated killing because the facts state that he acted only with the intent to injure. (B) is incorrect because, while assault with a deadly weapon may be a dangerous felony, the felony murder rule can be applied only when the underlying felony is independent of the killing. A felony such as assault or battery that directly causes death (as in this case) would not be considered an independent felony. (D) is incorrect because the enforcer can be convicted of the more serious crime of third degree murder, because he acted with the intent to cause serious bodily injury and death occurred. This satisfies the malice aforethought requirement for common law murder.

A plaintiff was injured when the steering mechanism of a snowmobile failed. He brought a negligence action against the snowmobile manufacturer. The steering mechanism was designed and manufactured by a component manufacturer; the snowmobile manufacturer merely assembled the snowmobile, branded it, and distributed it directly to retailers. To prevail against the snowmobile manufacturer, what will the plaintiff need to prove? A. That the steering mechanism was in a defective condition unreasonably dangerous to users. B. That the steering mechanism was in a defective condition unreasonably dangerous to users, and the plaintiff was the purchaser of the snowmobile, a member of the purchaser's family, or a guest of the purchaser. C. That the steering mechanism was in a defective condition unreasonably dangerous to users, and the defect could have been discovered and corrected if the component manufacturer had exercised reasonable care in its quality control process. D. That the steering mechanism was in a defective condition unreasonably dangerous to users, and the snowmobile manufacturer failed to inspect the mechanism before assembly of the snowmobile.

(C) The plaintiff will prevail against the snowmobile manufacturer if the component manufacturer was negligent in not discovering and correcting the defect. To establish a prima facie case of negligence in a products liability case, the plaintiff must show: (i) the existence of a legal duty owed by the defendant to that particular plaintiff; (ii) breach of that duty; (iii) actual and proximate cause; and (iv) damages. The duty of care arises when the defendant acts as a commercial supplier of products. A commercial supplier who assembles a product from components manufactured by others is subject to the same liability as the manufacturer of the defective component. To prove breach of duty, the plaintiff must show (i) negligent conduct by the defendant that leads to (ii) the supplying of a defective product. Here, because the snowmobile manufacturer assembled the snowmobile from component parts, including the steering mechanism manufactured by the component manufacturer, and sold the snowmobile as its own product, it will be liable for the negligence of the component manufacturer. Hence, if the steering mechanism was defective and the component manufacturer could have discovered and corrected the defect in the exercise of reasonable care, as (C) states, the snowmobile manufacturer has breached its duty to the plaintiff, and this breach of duty caused the plaintiff's injuries.

A retailer of personal watercraft agreed to sell to a buyer a speedboat for $10,000. The written contract specified delivery within 30 days and a down payment of $2,000, but did not contain a liquidated damages clause. Two weeks after making the down payment, the buyer told the retailer that he could not afford to go through with the purchase, and asked for his down payment back. The retailer, which could get as many of that model of speedboat as it required from the manufacturer for a wholesale price of $7,000, put the boat back in its inventory. The retailer then sold it to someone else for $10,500. The buyer sues the retailer to get back his deposit; the retailer counterclaims for damages. Excluding incidental costs, which of the following amounts represents the most likely recovery? A. The buyer will recover $2,000. B. The retailer will recover $500. C. The retailer will recover $1,000 D. The retailer will recover $2,500.

(C) The retailer will recover $1,000, which is the difference between its lost profits and the buyer's down payment. When the buyer repudiates or refuses to accept goods, the usual measure of the seller's damages is the difference between the contract price and the market price or the difference between the contract price and the resale price of the particular goods. However, neither of those measures of damages gives adequate compensation for the buyer's breach where the seller can obtain or manufacture as many goods as it can sell (i.e., a lost volume seller), because, but for the buyer's breach, the seller would have made two sales instead of one. In this type of case, lost profit is measured by the contract price less the cost to the dealer. Here, the retailer could have made two sales of that model of watercraft because it could get as many as it needed from the manufacturer. Hence, it lost a profit of $3,000 as a result of the buyer's breach. This amount is offset against the amount of the down payment that the buyer made, resulting in a net recovery of $1,000 by the retailer.

Recently enacted legislation required farmers in certain counties of a western state to use drip irrigation systems instead of traditional methods in order to conserve water for agricultural and other uses. A farmer who refused to use the drip system was charged pursuant to the enforcement provisions of the legislation. A state court enjoined him from using other irrigation methods and fined him. The farmer appealed to the state supreme court, renewing his trial court claims that the irrigation legislation violated a state constitutional provision prohibiting certain governmental intrusions into private commercial activities and that it was preempted by federal water management statutes. The state supreme court held that the state constitution prohibited the challenged legislation, and construed the relevant statutes as being within the parameters of the federal statutes, and thus preempted. If the state petitions for certiorari to the United States Supreme Court, how should the Court rule on the petition? A. Grant the petition, to determine whether the state court's interpretation of the scope of the federal statutes was incorrect. B. Grant the petition, because, under principles of federalism, a state court cannot be the final arbiter of the validity of its own legislation when it is alleged to be in conflict with federal law. C. Deny the petition, because there is no substantial federal question that is dispositive of the case. D. Deny the petition, because a state government may not seek review of decisions of its own courts in the United States Supreme Court.

(C) The state's challenge to the decision of the state supreme court presents no substantial federal question and will be denied. The Supreme Court's appellate jurisdiction under 28 U.S.C. section 1257 extends to reviewing the decision of the highest court of a state where the validity of state legislation is called into question on the ground that it is unconstitutional or contrary to federal statutes. However, the Supreme Court will hear a case from a state court only if the state court judgment turned on federal grounds. The Court will refuse jurisdiction if it finds adequate and independent nonfederal grounds to support the state decision, because a different interpretation of the federal statutes would have no effect on the judgment rendered by the state court, so that the Supreme Court, in effect, would be rendering an advisory opinion. Here, even if the state court was incorrect in holding that federal statutes preempted the state legislation, it also held that the state constitution prohibited the state legislation. Hence, a different interpretation of the federal preemption issue would have no effect on the outcome of the case.

The defendant, a competitive athlete, was charged with the murder of another athlete against whom she was scheduled to compete in two weeks. Autopsy results revealed that the victim was poisoned with a lethal mixture containing a variety of substances. During the prosecution's case in chief, evidence was introduced establishing that a bottle of a particular drug, which was among the substances listed in the autopsy report, was discovered in the defendant's medicine cabinet when she was arrested. On direct examination by her own attorney, the defendant states that when she was arrested and the bottle of the drug was found, she told the officers, "My doctor prescribed that for me to cope with the excruciating back pain from which I suffer." If the prosecution moves to strike this testimony, how should the court rule? A. For the defendant, because it is a prior consistent statement. B. For the defendant, because it tends to explain prosecution evidence. C. For the prosecution, because it is hearsay not within an exception. D. For the prosecution, because it is a self-serving statement.

(C) This question illustrates that a hearsay problem can arise even when the out-of-court declarant and the in-court witness are the same person. The defendant is attempting to testify as to a statement made by her out of court, and this statement is being offered to prove the truth of the matter asserted. Thus, the statement is hearsay. Because it is not within any exception to the hearsay rule, it must be excluded. Hearsay is a statement, other than one made by the declarant while testifying at the trial or hearing, offered in evidence to prove the truth of the matter asserted. [Fed. R. Evid. 801(c)] The defendant's out-of-court statement is being offered to prove the truth of the matter asserted therein, i.e., that she used the drug for her back pain. The defendant is free to make that assertion as part of her in-court testimony, but cannot use her out-of-court statement for that purpose. Thus, the statement is hearsay.

To encourage minority business and foster pride in minority heritage, a state adopted legislation exempting magazines and other periodicals from the state's receipts tax if 20% of the magazine is devoted to articles concerning minorities (a commission was set up to sample magazines to determine on a yearly basis whether they should be exempt). A publisher produced a sports magazine in the state that occasionally contained articles about minority athletes, but the commission determined that the publisher's magazine was not eligible for the receipts tax exemption. After paying the tax assessed on her magazine, the publisher sued for a refund. How will the court most likely rule? A. Against the publisher, because taxpayers do not have standing to challenge tax exemptions. B. Against the publisher, because the state has a compelling interest in encouraging minority business. C. In favor of the publisher, because the tax violates the Equal Protection Clause. D. In favor of the publisher, because the tax violates the First Amendment freedoms of speech and press.

(D) The court should rule in favor of the publisher because the tax exemption regulates speech based on its content in violation of the First Amendment. The freedom of the press is guaranteed by the First Amendment. As with other areas within the First Amendment, the freedom does not prohibit all government regulation of the press, but it does place limits on regulation. The press and broadcasting companies can be subject to general business regulations and taxes, but generally may not be singled out for a special tax. Moreover, a tax impacting on the press or a subpart of the press cannot be based on the content of the publication absent a compelling justification. Although the state tax here appears to be a general receipts tax, the exemption is based on content, which means that the tax also is based on content (i.e., a publication is subject to the tax unless it contains . . . ). As discussed below, a compelling interest is not presented here, so the exemption is invalid and the tax should fail.

A plumbing contractor sued a homeowner, alleging that the homeowner refused to pay for extensive pipe repairs performed on her home by an employee of the contractor. The contractor called the employee to the stand as a witness. The employee, under oath, testified that he did not perform any work at the homeowner's home. The employee also denied writing a letter to a friend telling the friend that he was going to do plumbing work on the homeowner's house. Without releasing the employee as a witness, the contractor offers in evidence the letter written by the employee to his friend. Which of the following is NOT a proper basis for admitting the employee's letter? A. Testimony by the employee's wife that she recognizes the employee's handwriting. B. The friend's testimony that the statements in the letter are responsive to a prior letter from the friend to the employee. C. Comparison by the jury of the letter with another letter that the employee has admitted writing. D. In-court comparison by the friend, a nonexpert, of the letter with another letter that the employee has admitted writing.

(D) The friend's comparison of the letters is not a proper basis for authenticating the employee's letter. Before a writing may be received into evidence, the writing must be authenticated by proof showing that the writing is what the proponent claims it is. The Federal Rules list several examples of proper methods of authentication through evidence of the genuineness of the handwriting of a letter writer. None of them, however, would permit an in-court comparison by the friend of the contested letter with another letter established to be the employee's. Under Federal Rule of Evidence 901(b), an expert witness or the trier of fact can determine the genuineness of a writing by comparing the questioned writing with another writing proved to be genuine. Thus, (C) would be a proper basis for admitting the employee's letter, because the jury, as the trier of fact, can compare the letter with another letter that the employee has admitted writing. The friend, however, cannot undertake that comparison because he is not an expert witness. While Rule 901 does not limit the methods of authentication, the rule governing opinion testimony by lay witnesses [Fed. R. Evid. 701] would preclude the friend's in-court comparison. To be admissible, opinion testimony by lay witnesses must be (i) rationally based on the perception of the witness; (ii) helpful to a clear understanding of his testimony or to determination of a fact in issue; and (iii) not based on scientific, technical, or other specialized knowledge. Here, the friend can add nothing to the jury's determination of the authenticity of the letter because the jury can compare for itself that letter with a letter the employee has admitted writing. Because the friend's in-court comparison would not be helpful to the jury here, it would not be a proper basis for authenticating the employee's letter. (A) is incorrect because a lay witness who has personal knowledge of the handwriting of the supposed writer may state her opinion as to whether the document is in that person's handwriting. Therefore, the employee's wife could properly authenticate the letter by testifying that she recognizes his handwriting. (B) is incorrect because the friend's testimony is proper circumstantial evidence of authentication under the reply letter doctrine. Under this doctrine, a letter may be authenticated by evidence that it was written in response to a communication sent to the claimed author. The content of the letter must make it unlikely that it was written by anyone other than the claimed author of the writing. Here, the employee's letter to the friend can be properly authenticated by the friend's testimony that it was a reply to a letter that he sent to the employee.

A homeowner and her neighbor purchased adjoining parcels of property 20 years ago. During the summer months, the homeowner ran electrical wires from her home to a guest house across land she knew belonged to the neighbor. The neighbor orally consented to the wiring's crossing his land. Two years ago, the neighbor sold his property to a purchaser. The following summer, the homeowner tried to run the wires across the purchaser's land, but the purchaser objected. The statute of limitations in ejectment is 15 years. If the homeowner files suit, how is the court likely to rule with respect to the land over which the electrical wires were laid? A. The homeowner has acquired title by adverse possession. B. The homeowner has acquired a prescriptive easement. C. The homeowner cannot claim any right on title because her use of the land in question was not continuous. D. The homeowner cannot claim any right on title because the neighbor consented to her use of the land for the wires.

(D) The homeowner cannot claim any right on title to the land over which the electrical wires were laid. The homeowner has not acquired a prescriptive easement because the neighbor consented to the wiring crossing his land. The holder of an easement has the right to use a tract of land for a special purpose, but has no right to possess and enjoy the tract of land. The neighbor's oral consent to the wiring's crossing his land was not effective to create an easement. The Statute of Frauds requires that any conveyance of an easement interest of greater than one year in duration must be in writing to be enforceable; an oral attempt to create an easement results in a revocable license. What the neighbor's consent did, however, was to make it impossible for the homeowner to acquire an easement by prescription. As with adverse possession, an easement by prescription requires that the use be open and notorious, adverse, and continuous and uninterrupted for the statutory period. Because the neighbor consented to the homeowner's running of the wires, the homeowner's use of the land was not adverse, and thus did not ripen into a prescriptive easement before the neighbor's conveyance to the purchaser.

A security officer employed by a mall was patrolling the mall parking lot which had suffered numerous thefts from cars when she heard a car alarm go off. She then saw a teenager stand up from behind the car. She immediately stopped the teenager and asked him what he was doing behind the car, and he said he was tying his shoe. Reasonably suspecting that he may have been trying to break into the car, she asked him to wait in the back seat of her car. The teenager complied, and waited in the back of the car while the security officer ate her lunch in the front seat and made a variety of personal calls. The teenager was humiliated because several of his friends and their parents, as well as some neighbors, saw him sitting in the security officer's car. Finally, after about an hour, the officer let the teenager go and advised him not to loiter in the parking lot anymore. If the teenager brings a false imprisonment action against the mall, will he be able to recover for the humiliation that he felt on sitting in the security officer's car in view of his friends and neighbors? A. No, humiliation is not actionable. B. No, because the security officer reasonably suspected that he was trying to break into a car. C. Yes, if the jury determines that the security officer's conduct was extreme and outrageous. D. Yes, because he was falsely imprisoned.

(D) The teenager will be able to recover for his humiliation because he was falsely imprisoned. All of the elements of a prima facie case for false imprisonment are present in these facts: an act or omission by defendant that confined or restrained plaintiff to a bounded area, intent by the defendant to do so, and causation. Here, the confinement was brought about by the invalid use of legal authority by the security officer. The mall cannot avail itself of the shopkeepers' privilege for detaining a suspected shoplifter because the detention must be for only a reasonable period of time for the purpose of making an investigation, and here the hour-long detention clearly was unreasonable given the officer's failure to even make an investigation during the detention. On proof of the prima facie case, the plaintiff can recover all foreseeable damages that arise from the tort. Humiliation is a foreseeable consequence of a false arrest, so the teenager will be able to recover. (A) is incorrect. While humiliation is not an actionable tort in and of itself, humiliation is a recognized element of damages from the commission of an intentional tort.

An owner purchased a parcel of property adjoining a five-foot-wide strip, which was a private right-of-way. Unsure where the exact boundaries of her property were located, the owner planted a garden on the five-foot right-of-way strip and enclosed it with a wire fence two weeks after taking up occupancy. The owner maintained the fence and garden for 20 years, at which time she removed the fence and smoothed out the ground where the garden had been located. Five years later, the owner entered into a written contract to sell the property to a buyer. The description in the contract included the five-foot strip. After research in the county recorder's office, the buyer discovered that the strip was a private right-of-way when the owner purchased the property. After properly notifying the owner of the problem prior to closing, the buyer refused to tender the purchase money to the owner when the closing day arrived. The owner sued the buyer for specific performance of the real estate sales contract. The jurisdiction's statutory adverse possession period is 15 years. Who will prevail? A. The buyer, because the owner failed to provide a marketable title. B. The buyer, because the owner surrendered her adverse possession rights when she removed the fence, as her possession was no longer open, notorious, and continuous. C. The buyer, because one may not adversely possess a right-of-way. D. The owner, because she held the right-of-way for a longer time than the minimum required by the state adverse possession statute.

(a) Absent a judgment in an action to quiet title or other tangible proof that title to the five-foot strip has actually been acquired, most jurisdictions would not consider the owner's title marketable. All contracts for the sale of land contain, unless the contract expressly provides otherwise, an implied covenant by the seller that she will deliver to the buyer a marketable title at the date of closing. Marketability refers to freedom from the possibility of litigation concerning the title; title is marketable if a reasonably prudent buyer, ready and able to purchase, will accept it in the exercise of ordinary prudence. At times, sellers will rely on adverse possession to show that defects in title have been cleared. However, courts generally will not permit such reliance when proof of adverse possession rests only on oral evidence that will not be available to the buyer in the future. Here, title to the property described in the contract is unmarketable because the five-foot strip was a private right-of-way and not part of the owner's record title. The owner's adverse possession of the strip will not be sufficient by itself to establish marketable title; there is no longer any physical evidence of the owner's possession. Thus, at the least the owner must offer the buyer additional proof that the buyer can use to defend any lawsuit challenging title.

A nephew asked his uncle, who like him was a farmer, to guarantee a loan to buy a new tractor. The local bank had already refused to extend credit to the nephew alone to buy the tractor. The uncle was inclined to refuse, but then decided that he could benefit from his own use of the tractor, so he told his nephew that he would guarantee the loan if he could use the new tractor without cost for 10 days during his harvest season. The nephew agreed to his uncle's proposal. The uncle went to the bank and told the loan officer that he was willing to guarantee the proposed loan to his nephew. This prompted the loan officer to agree to extend the requested credit to the nephew. Although the loan officer did not make the uncle sign any papers, the uncle provided consideration and the bank issued the nephew a loan commitment statement. That evening, the uncle had a change of heart. The next day, he telephoned the loan officer and told him to forget about his guaranteeing any loan to his nephew. Despite the uncle's phone call, the loan officer did not stop the check from being issued, and the nephew received the money to purchase the tractor. He drove the tractor over to the uncle's farm and delivered it for the uncle's 10-day use, as promised. The uncle told his nephew that he did not want to use the tractor and that he was not guaranteeing his loan. Within six months, it became clear that the nephew could not make good on the loan. If the bank sues the uncle for the unpaid portion of the loan, who will likely win? A. The bank, because the suretyship agreement was supported by consideration between the bank and the uncle. B. The bank, because the uncle's main purpose in making the agreement with the bank was to benefit himself, not his nephew. C. The uncle, because the suretyship agreement was not in writing. D. The uncle, because he withdrew his promise before the nephew received the money or the tractor.

(b)

Owen, the owner of Greenacre, sold it to Alice for $100,000. Alice did not record the deed, and left the country on an extended trip. Owen, seeing an opportunity to make a quick profit, partitioned Greenacre and sold the front half, Frontacre, to Bert in exchange for $50,000. Bert, who knew nothing about Alice's interest in the property, promptly recorded his interest. Two months later, Bert sold Frontacre to Carl in exchange for $55,000. Carl was aware of Alice's interest in Frontacre but recorded his deed to Frontacre anyway. Meanwhile, Owen executed a mortgage on the back half of the property, Backacre, to Bank in the amount of $40,000. Bank knew nothing of Owen's transaction with Alice but neglected to record its mortgage interest. Six months later, Alice returned home and recorded her deed to Greenacre. A statute in the jurisdiction provides: "Any conveyance of an estate in land, other than a lease for less than one year, shall not be valid against any subsequent purchaser for value, without notice, unless the conveyance is recorded." If Alice brings an action to quiet title in Greenacre, how is a court likely to classify her claim? A. As superior to Owen's rights in Backacre and Carl's rights in Frontacre, and not subject to Bank's mortgage on Backacre. B. As superior to Owen's rights in Backacre, inferior to Carl's rights in Frontacre, and subject to Bank's mortgage on Backacre. C. As superior to Owen's rights in Backacre, inferior to Carl's rights in Frontacre, and not subject to Bank's mortgage on Backacre. D. As superior to Owen's rights in Backacre and Carl's rights in Frontacre, but subject to Bank's mortgage on Backacre.

(b) Alice has title to Backacre subject to Bank's mortgage, and Alice has no rights in Frontacre. The recording statute in the question is a pure notice statute, which allows subsequent purchasers for value and without notice of a prior conveyance to prevail over the prior transferee, regardless of whether the subsequent purchaser records. In addition, the "shelter rule" allows a person who takes from a bona fide purchaser to prevail against any interest that the transferor-bona fide purchaser would have prevailed against, even if the transferee had actual knowledge of the prior unrecorded interest. Thus, Carl would prevail over Alice even though Carl was aware of Alice's interest in the parcel, because Carl obtained title from Bert, a bona fide purchaser. Thus, Alice's rights in Frontacre are extinguished. With regard to Backacre, Alice has superior title to it over Owen, but takes subject to the mortgage on it by Bank because mortgagees for value are treated as "purchasers" under recording statutes and because Bank had no notice of Alice's interest.

A nephew asked his uncle, who like him was a farmer, to guarantee a loan to buy a new tractor. The local bank had already refused to extend credit to the nephew alone to buy the tractor. The uncle was inclined to refuse, but then decided that he could benefit from his own use of the tractor, so he told his nephew that he would guarantee the loan if he could use the new tractor without cost for 10 days during his harvest season. The nephew agreed to his uncle's proposal. The uncle went to the bank and told the loan officer that he was willing to guarantee the proposed loan to his nephew. This prompted the loan officer to agree to extend the requested credit to the nephew. Although the loan officer did not make the uncle sign any papers, the uncle provided consideration and the bank issued the nephew a loan commitment statement. That evening, the uncle had a change of heart. The next day, he telephoned the loan officer and told him to forget about his guaranteeing any loan to his nephew. Despite the uncle's phone call, the loan officer did not stop the check from being issued, and the nephew received the money to purchase the tractor. He drove the tractor over to the uncle's farm and delivered it for the uncle's 10-day use, as promised. The uncle told his nephew that he did not want to use the tractor and that he was not guaranteeing his loan. Within six months, it became clear that the nephew could not make good on the loan. If the bank sues the uncle for the unpaid portion of the loan, who will likely win? A. The bank, because the suretyship agreement was supported by consideration between the bank and the uncle. B. The bank, because the uncle's main purpose in making the agreement with the bank was to benefit himself, not his nephew. C. The uncle, because the suretyship agreement was not in writing. D. The uncle, because he withdrew his promise before the nephew received the money or the tractor.

(b) The uncle's main purpose in making the agreement was to benefit himself rather than his nephew, so the agreement is outside of the Statute of Frauds and is enforceable even though it was oral. Under the Statute of Frauds, certain agreements must be evidenced by a writing that contains: (i) the identity of the party sought to be charged; (ii) identification of the contract's subject matter; (iii) terms and conditions of the agreement; (iv) recital of consideration; and (v) signature of the party to be charged, or of his agent. One type of agreement that is covered by the Statute of Frauds is a promise to answer for the debt or default of another where the promise is collateral rather than primary. However, where the main purpose or leading object of the promisor is to secure an advantage or pecuniary benefit for himself, the contract is not within the Statute of Frauds, even if the effect is still to pay the debt of another. The uncle guaranteed the loan to his nephew, which means that the uncle agreed to repay the loan only if his nephew refused to do so. Therefore, the uncle made a collateral promise to answer for the debt or default of the nephew regarding the loan from the bank. However, because the main purpose of the uncle's making the agreement was to benefit himself rather than his nephew (by, e.g., being allowed to use the tractor without cost), the agreement is outside the scope of the Statute of Frauds and would be enforceable against the uncle even in the absence of a writing.


Conjuntos de estudio relacionados

Chapter 13: Theatre Today: Diverse and Global

View Set

Climate vs. Weather/Climate & Latitude

View Set

Mike Holy Shit That Was an Unnecessarily Large Assignment 250

View Set

Implementing Security in the Systems and Software Development Lifecycle

View Set

Kress- Entrepreneurship/ small business test 2

View Set

Unit 9: Lesson 1 - Organic Chemistry

View Set

FIN 443 Enterprise Risk Management: Module 4 Study Guide Questions

View Set